You are on page 1of 103

Putnam In Olympiad


Mahdiyar Amid Sardari
October 6, 2022

Introduction
First let talk about Mr.Putnam; William Lowell Putnam II (November 22, 1861–
June 1923) (more commonly known as William Putnam, Sr.) was an American
lawyer and banker.Putnam was the son of George and Harriet (Lowell) Putnam.
He graduated from Harvard in 1882, and proceeded to make a professional name
for himself in legal and financial circles
Now let talk about Putnam’s competition; The William Lowell Putnam Math-
ematical Competition, often abbreviated to Putnam Competition, is an annual
mathematics competition for undergraduate college students enrolled at institu-
tions of higher learning in the United States and Canada (regardless of the stu-
dents’ nationalities). It awards a scholarship and cash prizes ranging from $250
to $2, 500 for the top students and $5, 000 to $25, 000 for the top schools, plus
one of the top five individual scorers (designated as Putnam Fellows) is awarded
a scholarship of up to 12, 000 plus tuition at Harvard University (Putnam Fellow
Prize Fellowship), the top 100 individual scorers have their names mentioned in
the American Mathematical Monthly (alphabetically ordered within rank), and
the names and addresses of the top 500 contestants are mailed to all participat-
ing institutions. It is widely considered to be the most prestigious university-
level mathematics competition in the world, and its difficulty is such that the
median score is often zero (out of 120) despite being attempted by students spe-
cializing in mathematics. The competition was founded in 1927 by Elizabeth
Lowell Putnam in memory of her husband William Lowell Putnam, who was an
advocate of intercollegiate intellectual competition. The competition has been
offered annually since 1938 and is administered by the Mathematical Associa-
tion of America.
As you read Putnam is one of the oldest competition in the world and also with
a high quality. So I decided to collect all problems from Putnam that can be
solved by 1 Olympiad literacy.
∗ If you have any idea or find any mistake, you can keep in touch with me:

mahdiyaramid2004@gmail.com.
1 Actually Olympiad literacy means the tools that we need to solve an Olympiad problems,

specifically IMO.

1
Putnam In Olympiad Mahdiyar Amid Sardari

Problems

2021
(A1) A grasshopper starts at the origin in the coordinate plane and makes a
sequence of hops. Each hop has length 5, and after each hop the grasshopper
is at a point whose coordinates are both integers; thus, there are 12 possible
locations for the grasshopper after the first hop. What is the smallest number
of hops needed for the grasshopper to reach the point (2021, 2021)?

(A3) Determine all positive integers N for which the sphere


x2 + y 2 + z 2 = N
has an inscribed regular tetrahedron whose vertices have integer coordinates.

(A5) Let A be the set of all integers n such that 1 ≤ n ≤ 2021 and gcd(n, 2021) =
1. For every non negative integer j, let
X
S(j) = nj .
n∈A

Determine all values of j such that S(j) is a multiple of 2021.

(B2) Determine the maximum value of the sum



X n 1
S= n
(a1 a2 . . . an ) n
n=1
2

over all sequences a1 , a2 , a3 , . . . of non negative real numbers satisfying



X
ak = 1.
k=1

(B4) Let F0 , F1 , . . . be the sequence of Fibonacci numbers, with F0 = 0, F1 = 1,


and Fn = Fn−1 + Fn−2 for n ≥ 2. For m > 2, let Rm be the remainder when
QFm −1 k
the product k=1 k is divided by Fm . Prove that Rm is also a Fibonacci
number.

2020
(A1) How many positive integers N satisfy all of the following three conditions?
(1) N is divisible by 2020.

Page 2
Putnam In Olympiad Mahdiyar Amid Sardari

(2) N has at most 2020 decimal digits.


(3) The decimal digits of N are a string of consecutive ones followed by a string
of consecutive zeros.

(A2) Let k be a non negative integer. Evaluate:


k  
X k+j
2k−j .
j=0
j

(A3) Let a0 = π/2, and let an = sin(an−1 ) for n ≥ 1. Determine whether



X
a2n
n=1
converges.

(A4) Consider a horizontal strip of N + 2 squares in which the first and the
last square are black and the remaining N squares are all white. Choose a
white square uniformly at random, choose one of its two neighbors with equal
probability, and color tis neighboring square black if it is not already black. Re-
peat this process until all the remaining white squares have only black neighbors.
Let w(N ) be the expected number of white squares remaining. Find
w(N )
lim .
N →∞ N

(A5) Let an be the number of sets S of positive integers for which


X
Fk = n,
k∈S

where the Fibonacci sequence (Fk )k≥1 satisfies Fk+2 = Fk+1 + Fk and begins
F1 = 1, F2 = 1, F3 = 2, F4 = 3. Find the largest number n such that an = 2020.

(A6) For a positive integer N , let fN be the function defined by


N
X N + 1/2 − n
fN (x) = sin ((2n + 1)x) .
n=0
(N + 1)(2n + 1)

Determine the smallest constant M such that fN (x) ≤ M for all N and all real x.

(B1) For a positive integer n, define d(n) to be the sum of the digits of n
when written in binary (for example, d(13) = 1 + 1 + 0 + 1 = 3). Let
2020
X
S= (−1)d(k) k 3 .
k=1

Page 3
Putnam In Olympiad Mahdiyar Amid Sardari

Determine S modulo 2020.

(B2) Let k and n be integers with 1 ≤ k < n. Alice and Bob play a game
with k pegs in a line of n holes. At the beginning of the game, the pegs occupy
the k leftmost holes. A legal move consists of moving a single peg to any vacant
hole that is further to the right. The players alternate moves, with Alice playing
first. The game ends when the pegs are in the k rightmost holes, so whoever is
next to play cannot move and therefore loses. For what values of n and k does
Alice have a winning strategy?

(B4) Let n be a positive integer, and let Vn be the set of integer (2n + 1)-
tuples v = (s0 , s1 , · · · , s2n−1 , s2n ) for which s0 = s2n = 0 and |sj − sj−1 | = 1
for j = 1, 2, · · · , 2n. Define
2n−1
X
q(v) = 1 + 3sj ,
j=1

1
and let M (n) be the average of q(v) over all v ∈ Vn . Evaluate M (2020).

(B5) For j ∈ {1, 2, 3, 4}, let zj be a complex number with |zj | = 1 and zj ̸= 1.
Prove that
3 − z1 − z2 − z3 − z4 + z1 z2 z3 z4 ̸= 0.

(B6) Let n be a positive integer. Prove that


n
X √
(−1)⌊k( 2−1)⌋
≥ 0.
k=1

(As usual, ⌊x⌋ denotes the greatest integer less than or equal to x.)

2019
(A1) Determine all possible values of A3 + B 3 + C 3 − 3ABC where A, B, and
C are non negative integers.

(A2) In the triangle △ABC, let G be the centroid, and let I be the center of the
inscribed circle. Let α and β be the angles at the vertices A and B, respectively.
Suppose that the segment IG is parallel to AB and that β = 2 tan−1 (1/3). Find
α.

(A3) Given real numbers b0 , b1 , . . . , b2019 with b2019 ̸= 0, let z1 , z2 , . . . , z2019

Page 4
Putnam In Olympiad Mahdiyar Amid Sardari

be the roots in the complex plane of the polynomial


2019
X
P (z) = bk z k .
k=0

Let µ = (|z1 |+· · ·+|z2019 |)/2019 be the average of the distances from z1 , z2 , . . . , z2019
to the origin. Determine the largest constant M such that µ ≥ M for all choices
of b0 , b1 , . . . , b2019 that satisfy

1 ≤ b0 < b1 < b2 < · · · < b2019 ≤ 2019.

(B1) Denote by Z 2 the set of all points (x, y) in the plane with integer co-
ordinates. For each integer n ≥ 0, let Pn be the subset of Z 2 consisting of
the point (0, 0) together with all points (x, y) such that x2 + y 2 = 2k for some
integer k ≤ n. Determine, as a function of n, the number of four-point subsets
of Pn whose elements are the vertices of a square.
(2k−1)π
Pn−1 sin( 2n ) an
(B2) For all n ≥ 1, let an = k=1 cos2 ( (k−1)π ) cos2 ( kπ ) . Determine limn→∞ n3 .
2n 2n

(B5) Let Fm be the m’th Fibonacci number, defined by F1 = F2 = 1 and


Fm = Fm−1 + Fm−2 for all m ≥ 3. Let p(x) be the polynomial of degree 1008
such that p(2n + 1) = F2n+1 for n = 0, 1, 2, . . . , 1008. Find integers j and k such
that p(2019) = Fj − Fk .

(B6) Let Z n be the integer lattice in Rn . Two points in Z n are called neighbors if
they differ by exactly 1 in one coordinate and are equal in all other coordinates.
For which integers n ≥ 1 does there exist a set of points S ⊂ Z n satisfying the
following two conditions? (1) If p is in S, then none of the neighbors of p is in
S. (2) If p ∈ Z n is not in S, then exactly one of the neighbors of p is in S.

2018
(A1) Find all ordered pairs (a, b) of positive integers for which
1 1 3
+ = .
a b 2018

P10
(A3) Determine the greatest possible value of i=1 cos(3xi ) for real numbers
P10
x1 , x2 , . . . , x10 satisfying i=1 cos(xi ) = 0.

Page 5
Putnam In Olympiad Mahdiyar Amid Sardari

(A4) Let m and n be positive integers with gcd(m, n) = 1, and let


   
mk m(k − 1)
ak = −
n n
for k = 1, 2, . . . , n. Suppose that g and h are elements in a group G and that

gha1 gha2 · · · ghan = e,

where e is the identity element. Show that gh = hg. (As usual, ⌊x⌋ denotes the
greatest integer less than or equal to x.)

(A5) Let f : R → R be an infinitely differentiable function satisfying f (0) = 0,


f (1) = 1, and f (x) ≥ 0 for all x ∈ R. Show that there exist a positive integer
n and a real number x such that f (n) (x) < 0.

(A6) Suppose that A, B, C, and D are distinct points, no three of which lie on
a line, in the Euclidean plane. Show that if the squares of the lengths of the
line segments AB, AC, AD, BC, BD, and CD are rational numbers, then the
quotient
area(△ABC)
area(△ABD)
is a rational number.

(B1) Let P be the set of vectors defined by


  
a
P= 0 ≤ a ≤ 2, 0 ≤ b ≤ 100, and a, b ∈ Z .
b

Find all v ∈ P such that the set P \ {v} obtained by omitting vector v from P
can be partitioned into two sets of equal size and equal sum.

(B3) Find all positive integers n < 10100 for which simultaneously n divides
2n , n − 1 divides 2n − 1, and n − 2 divides 2n − 2.

(B4) Given a real number a, we define a sequence by x0 = 1, x1 = x2 = a,


and xn+1 = 2xn xn−1 − xn−2 for n ≥ 2. Prove that if xn = 0 for some n, then
the sequence is periodic.

(B6) Let S be the set of sequences of length 2018 whose terms are in the set
{1, 2, 3, 4, 5, 6, 10} and sum to 3860. Prove that the cordiality of S is at most
 2018
2018
23860 · .
2048

Page 6
Putnam In Olympiad Mahdiyar Amid Sardari

2017
(A1) Let S be the smallest set of positive integers such that
(1) 2 is in S,
(2) n is in S whenever n2 is in S, and
(3) (n + 5)2 is in S whenever n is in S.
Which positive integers are not in S?

(A2) Let Q0 (x) = 1, Q1 (x) = x, and


(Qn−1 (x))2 − 1
Qn (x) =
Qn−2 (x)
for all n ≥ 2. Show that, whenever n is a positive integer, Qn (x) is equal to a
polynomial with integer coefficients.

(A4) A class with 2N students took a quiz, on which the possible scores were
0, 1, . . . , 10. Each of these scores occurred at least once, and the average score
was exactly 7.4. Show that the class can be divided into two groups of N stu-
dents in such a way that the average score for each group was exactly 7.4.

(A5) Each of the integers from 1 to n is written on a separate card, and then the
cards are combined into a deck and shuffled. Three players, A, B, and C, take
turns in the order A, B, C, A, . . . choosing one card at random from the deck.
(Each card in the deck is equally likely to be chosen.) After a card is chosen,
that card and all higher-numbered cards are removed from the deck, and the
remaining cards are reshuffled before the next turn. Play continues until one of
the three players wins the game by drawing the card numbered 1. Show that
for each of the three players, there are arbitrarily large values of n for which
that player has the highest probability among the three players of winning the
game.

(A6) The 30 edges of a regular icosahedron are distinguished by labeling them


1, 2, . . . , 30. How many different ways are there to paint each edge red, white, or
blue such that each of the 20 triangular faces of the icosahedron has two edges
of the same color and a third edge of a different color?

(B1) Let L1 and L2 be distinct lines in the plane. Prove that L1 and L2
intersect if and only if, for every real number λ ̸= 0 and every point P not on
−−→ −−→
L1 or L2 , there exist points A1 on L1 and A2 on L2 such that P A2 = λP A1 .

(B2) Suppose that a positive integer N can be expressed as the sum of k con-
secutive positive integers
N = a + (a + 1) + (a + 2) + · · · + (a + k − 1)
for k = 2017 but for no other values of k > 1. Considering all positive integers
N with this property, what is the smallest positive integer a that occurs in any

Page 7
Putnam In Olympiad Mahdiyar Amid Sardari

of these expressions?

(B3) Suppose that



X
f (x) = ci xi
i=0

is a power series for which each coefficient ci is 0 or 1. Show that if f (2/3) = 3/2,
then f (1/2) must be irrational.

(B5) A line in the plane of a triangle T is called an equalizer if it divides T


into two regions having equal area and equal perimeter. Find positive integers
a > b > c, with a as small as possible, such that there exists a triangle with side
lengths a, b, c that has exactly two distinct equalizers.

(B6) Find the number of ordered 64-tuples {x0 , x1 , . . . , x63 } such that x0 , x1 , . . . , x63
are distinct elements of {1, 2, . . . , 2017} and

x0 + x1 + 2x2 + 3x3 + · · · + 63x63

is divisible by 2017.

2016
(A2) Given a positive integer n, let M (n) be the largest integer m such that
 
m−1
mn − 1 > .
n
Evaluate
M (n)
lim .
n→∞ n

(A4) Consider a (2m − 1) × (2n − 1) rectangular region, where m and n are


integers such that m, n ≥ 4. The region is to be tiled using tiles of the two types
shown:

(The dotted lines divide the tiles into 1 × 1 squares.) The tiles may be rotated
and reflected, as long as their sides are parallel to the sides of the rectangular
region. They must all fit within the region, and they must cover it completely
without overlapping. What is the minimum number of tiles required to tile the
region?

Page 8
Putnam In Olympiad Mahdiyar Amid Sardari

(A5) Suppose that G is a finite group generated by the two elements g and
h, where the order of g is odd. Show that every element of G can be written in
the form
g m1 hn1 g m2 hn2 · · · g mr hnr
with 1 ≤ r ≤ |G| and mn , n1 , m2 , n2 , . . . , mr , nr ∈ {1, −1}. (Here |G| is the
number of elements of G.)

(B3) Suppose that S is a finite set of points in the plane such that the area
of triangle △ABC is at most 1 whenever A, B, and C are in S. Show that there
exists a triangle of area 4 that (together with its interior) covers the set S.

(B5) Find all functions f from the interval (1, ∞) to (1, ∞) with the follow-
ing property: if x, y ∈ (1, ∞) and x2 ≤ y ≤ x3 , then (f (x))2 ≤ f (y) ≤ (f (x))3 .

(B6) Evaluate:
∞ ∞
X (−1)k−1 X 1
n+1
.
k n=0
k2
k=1

2015
(A2) Let a0 = 1, a1 = 2, and an = 4an−1 − an−2 for n ≥ 2.
Find an odd prime factor of a2015 .

(A4) For each real number x, let


X 1
f (x) =
2n
n∈Sx

where Sx is the set of positive integers n for which ⌊nx⌋ is even. What is the
largest real number L such that f (x) ≥ L for all x ∈ [0, 1)?
(As usual, ⌊z⌋ denotes the greatest integer less than or equal to z.

(A5) Let q be an odd positive integer, and let Nq denote the number of in-
tegers a such that 0 < a < q/4 and gcd(a, q) = 1. Show that Nq is odd if and
only if q is of the form pk with k a positive integer and p a prime congruent to
5 or 7 modulo 8.

(B2) Given a list of the positive integers 1, 2, 3, 4, . . . , take the first three num-
bers 1, 2, 3 and their sum 6 and cross all four numbers off the list. Repeat with
the three smallest remaining numbers 4, 5, 7 and their sum 16. Continue in this
way, crossing off the three smallest remaining numbers and their sum and con-
sider the sequence of sums produced: 6, 16, 27, 36, . . . . Prove or disprove that

Page 9
Putnam In Olympiad Mahdiyar Amid Sardari

there is some number in this sequence whose base 10 representation ends with
2015.

(B4) Let T be the set of all triples (a, b, c) of positive integers for which there
exist triangles with side lengths a, b, c. Express
X 2a
3b 5c
(a,b,c)∈T

as a rational number in lowest terms.

(B5) Let Pn be the number of permutations π of {1, 2, . . . , n} such that

|i − j| = 1 implies |π(i) − π(j)| ≤ 2

for all i, j in {1, 2, . . . , n}. Show that for n ≥ 2, the quantity

Pn+5 − Pn+4 − Pn+3 + Pn

does not depend on n, and find its value.

(B6) For each positive


h √ integer
 k, let A(k) be the number of odd divisors of
k in the interval 1, 2k . Evaluate:


X A(k)
(−1)k−1 .
k
k=1

2014
(A1) Prove that every nonzero coefficient of the Taylor series of (1 − x + x2 )ex
about x = 0 is a rational number whose numerator (in lowest terms) is either 1
or a prime number.

(A3) Let a0 = 5/2 and ak = a2k−1 − 2 for k ≥ 1. Compute


∞  
Y 1
1−
ak
k=0

in closed form.

(A4) Suppose X is a random variable that takes on only non negative inte-
ger values, with E[X] = 1, E[X 2 ] = 2, and E[X 3 ] = 5. (Here E[Y ] denotes the
expectation of the random variable Y.) Determine the smallest possible value of

Page 10
Putnam In Olympiad Mahdiyar Amid Sardari

the probability of the event X = 0.

(A5) Let Pn (x) = 1 + 2x + 3x2 + · · · + nxn−1 . Prove that the polynomials


Pj (x) and Pk (x) are relatively prime for all positive integers j and k with j ̸= k.

(B1) A base 10 over-expansion of a positive integer N is an expression of the


form N = dk 10k + dk−1 10k−1 + · · · + d0 100 with dk ̸= 0 and di ∈ {0, 1, 2, . . . , 10}
for all i. For instance, the integer N = 10 has two base 10 over-expansions:
10 = 10 · 100 and the usual base 10 expansion 10 = 1 · 101 + 0 · 100 . Which
positive integers have a unique base 10 over-expansion?

(B4) Show that for each positive integer n, all the roots of the polynomial
n
X
2k(n−k) xk
k=0

are real numbers.

(B5) In the 75th Annual Putnam Games, participants compete at mathematical


games. Patniss and Keeta play a game in which they take turns choosing an
element from the group of inevitable n × n matrices with entries in the field
Z/pZ of integers modulo p, where n is a fixed positive integer and p is a fixed
prime number. The rules of the game are:
(1) A player cannot choose an element that has been chosen by either player on
any previous turn.
(2) A player can only choose an element that commutes with all previously cho-
sen elements.
(3) A player who cannot choose an element on his/her turn loses the game.
Patniss takes the first turn. Which player has a winning strategy?

2013
(A1) Recall that a regular icosahedron is a convex polyhedron having 12 vertices
and 20 faces; the faces are congruent equilateral triangles. On each face of a
regular icosahedron is written a non negative integer such that the sum of all
20 integers is 39. Show that there are two faces that share a vertex and have
the same integer written on them.

(A2) Let S be the set of all positive integers that are not perfect squares. For n
in S, consider choices of integers a1 , a2 , . . . , ar such that n < a1 < a2 < · · · < ar
and n·a1 ·a2 · · · ar is a perfect square, and let f (n) be the minimum of ar over all
such choices. For example, 2 · 3 · 6 is a perfect square, while 2 · 3, 2 · 4, 2 · 5, 2 · 3 · 4,
2 · 3 · 5, 2 · 4 · 5, and 2 · 3 · 4 · 5 are not, and so f (2) = 6. Show that the function

Page 11
Putnam In Olympiad Mahdiyar Amid Sardari

f from S to the integers is one-to-one.

(A3) Suppose that the real numbers a0 , a1 , . . . , an and x, with 0 < x < 1,
satisfy
a0 a1 an
+ + ··· + = 0.
1 − x 1 − x2 1 − xn+1
Prove that there exists a real number y with 0 < y < 1 such that
a0 + a1 y + · · · + an y n = 0.

(A4) A finite collection of digits 0 and 1 is written around a circle. An arc


of length L ≥ 0 consists of L consecutive digits around the circle. For each arc
w, let Z(w) and N (w) denote the number of 0’s in w and the number of 1’s in
w, respectively. Assume that |Z(w) − Z(w′ )| ≤ 1 for any two arcs w, w′ of the
same length. Suppose that some arcs w1 , . . . , wk have the property that
k k
1X 1X
Z= Z(wj ) and N = N (wj )
k j=1 k j=1

are both integers. Prove that there exists an arc w with Z(w) = Z and
N (w) = N.

(A5) For m ≥ 3, a list of m



3 real numbers aijk (1 ≤ i < j < k ≤ m) is
said to be area definite for Rn if the inequality
X
aijk · Area(△Ai Aj Ak ) ≥ 0
1≤i<j<k≤m

holds for every choice of m points A1 , . . . , Am in Rn . For example, the list of


four numbers a123 = a124 = a134 = 1, a234 = −1 is area definite for R2 . Prove
that if a list of m 2 3
3 numbers is area definite for R , then it is area definite for R .

(A6) Define a function w : Z × Z → Z as follows. For |a|, |b| ≤ 2, let w(a, b) be


as in the table shown; otherwise, let w(a, b) = 0.
b
w(a, b) −2 −1 0 1 2
−2 −1 −2 2 −2 −1
−1 −2 4 −4 4 −2
a 0 2 −4 12 −4 2
1 −2 4 −4 4 −2
2 −1 −2 2 −2 −1
For every finite subset S of Z × Z, define
X
A(S) = w(s − s′ ).
(s,s′ )∈S×S

Page 12
Putnam In Olympiad Mahdiyar Amid Sardari

Prove that if S is any finite nonempty subset of Z × Z, then A(S) > 0.

(B1) For positive integers n, let the numbers c(n) be determined by the rules
c(1) = 1, c(2n) = c(n), and c(2n + 1) = (−1)n c(n). Find the value of
2013
X
c(n)c(n + 2).
n=1

S∞
(B2) Let C = N =1 CN , where CN denotes the set of ’cosine polynomials’
of the form
N
X
f (x) = 1 + an cos(2πnx)
n=1

for which:
(1) f (x) ≥ 0 for all real x, and
(2) an = 0 whenever n is a multiple of 3.
Determine the maximum value of f (0) as f ranges through C, and prove that
this maximum is attained.

(B3) Let P be a nonempty collection of subsets of {1, . . . , n} such that:


(1) if S, S ′ ∈ P, then S ∪ S ′ ∈ P and S ∩ S ′ ∈ P, and
(2) if S ∈ P and S ̸= ∅, then there is a subset T ⊂ S such that T ∈ P and T
contains exactly one fewer element than S.
Suppose that f : P → R is a function such that f (∅) = 0 and

f (S ∪ S ′ ) = f (S) + f (S ′ ) − f (S ∩ S ′ ) for all S, S ′ ∈ P.

Must there exist real numbers f1 , . . . , fn such that


X
f (S) = fi
i∈S

for every S ∈ P ?

(B5) Let X = {1, 2, . . . , n}, and let k ∈ X. Show that there are exactly k · nn−1
functions f : X → X such that for every x ∈ X there is a j ≥ 0 such that
f (j) (x) ≤ k. [Here f (j) (0)
 denotes the jth iterate of f, so that f (x) = x and
(j+1) (j)
f (x) = f f (x) .]

(B6) Let n ≥ 1 be an odd integer. Alice and Bob play the following game,
taking alternating turns, with Alice playing first. The playing area consists of n
spaces, arranged in a line. Initially all spaces are empty. At each turn, a player
either
(1) places a stone in an empty space, or
(2) removes a stone from a nonempty space s, places a stone in the nearest
empty space to the left of s (if such a space exists), and places a stone in the

Page 13
Putnam In Olympiad Mahdiyar Amid Sardari

nearest empty space to the right of s (if such a space exists).


Furthermore, a move is permitted only if the resulting position has not occurred
previously in the game. A player loses if he or she is unable to move. Assuming
that both players play optimally throughout the game, what moves may Alice
make on her first turn?

2012
(A1) Let d1 , d2 , . . . , d12 be real numbers in the open interval (1, 12). Show that
there exist distinct indices i, j, k such that di , dj , dk are the side lengths of an
acute triangle.

(A2) Let ∗ be a commutative and associative binary operation on a set S. As-


sume that for every x and y in S, there exists z in S such that x ∗ z = y. (This z
may depend on x and y.) Show that if a, b, c are in S and a∗c = b∗c, then a = b.

 → Rbe a continuous function such that


(A3) Let f : [−1, 1]
2−x2 x2
(1) f (x) = 2 f 2−x2 for every x in [−1, 1],
(2) f (0) = 1, and
(3) limx→1− √f1−x
(x)
exists and is finite.
Prove that f is unique, and express f (x) in closed form.

(A4) Let q and r be integers with q > 0, and let A and B be intervals on
the real line. Let T be the set of all b + mq where b and m are integers with b in
B, and let S be the set of all integers a in A such that ra is in T. Show that if
the product of the lengths of A and B is less than q, then S is the intersection
of A with some arithmetic progression.

(A6) Let f (x, y) be a continuous, real-valued function on R2 . Suppose that,


for every rectangular region R of area 1, the double integral of f (x, y) over R
equals 0. Must f (x, y) be identically 0?

(B2) Let P be a given (non-degenerate) polyhedron. Prove that there is a


constant c(P ) > 0 with the following property: If a collection of n balls whose
volumes sum to V contains the entire surface of P, then n > c(P )/V 2 .

(B3) A round-robin tournament among 2n teams lasted for 2n − 1 days, as


follows. On each day, every team played one game against another team, with
one team winning and one team losing in each of the n games. Over the course
of the tournament, each team played every other team exactly once. Can one
necessarily choose one winning team from each day without choosing any team
more than once?

Page 14
Putnam In Olympiad Mahdiyar Amid Sardari

(B6) Let p be an odd prime number such that p ≡ 2 (mod 3). Define a permu-
tation π of the residue classes modulo p by π(x) ≡ x3 (mod p). Show that π is
an even permutation if and only if p ≡ 3 (mod 4).

2011
(A1) Define a growing spiral in the plane to be a sequence of points with integer
coordinates P0 = (0, 0), P1 , . . . , Pn such that n ≥ 2 and:
(1) The directed line segments P0 P1 , P1 P2 , . . . , Pn−1 Pn are in successive coor-
dinate directions east (for P0 P1 ), north, west, south, east, etc.
(2) The lengths of these line segments are positive and strictly increasing.
North

P3àb bàP2

West bà àb East


P0 P1

àb bà
P4 P5
South
How many of the points (x, y) with integer coordinates 0 ≤ x ≤ 2011, 0 ≤ y ≤
2011 cannot be the last point, Pn , of any growing spiral?

(A2) Let a1 , a2 , . . . and b1 , b2 , . . . be sequences of positive real numbers such


that a1 = b1 = 1 and bn = bn−1 an − 2 for n = 2, 3, . . . . Assume that the
sequence (bj ) is bounded. Prove that

X 1
S=
n=1
a1 · · · an

converges, and evaluate S.

(B1) Let h and k be positive integers. Prove that for every ε > 0, there are
positive integers m and n such that
√ √
ε < h m − k n < 2ε.

Page 15
Putnam In Olympiad Mahdiyar Amid Sardari

(B2) Let S be the set of all ordered triples (p, q, r) of prime numbers for which
at least one rational number x satisfies px2 + qx + r = 0. Which primes appear
in seven or more elements of S?

(B6) Let p be an odd prime. Show that for at least (p + 1)/2 values of n
in {0, 1, 2, . . . , p − 1},
p−1
X
k!nk is not divisible by p.
k=0

2010
(A1) Given a positive integer n, what is the largest k such that the numbers
1, 2, . . . , n can be put into k boxes so that the sum of the numbers in each box
is the same?
10n n
(A4) Prove that for each positive integer n, the number 1010 + 1010 + 10n − 1
is not prime.

(B1) Is there an infinite sequence of real numbers a1 , a2 , a3 , . . . such that

am m m
1 + a2 + a3 + · · · = m

for every positive integer m?

(B2) Given that A, B, and C are non collinear points in the plane with in-
teger coordinates such that the distances AB, AC, and BC are integers, what
is the smallest possible value of AB?

(B3) There are 2010 boxes labeled B1 , B2 , . . . , B2010 , and 2010n balls have been
distributed among them, for some positive integer n. You may redistribute the
balls by a sequence of moves, each of which consists of choosing an i and moving
exactly i balls from box Bi into any one other box. For which values of n is it
possible to reach the distribution with exactly n balls in each box, regardless of
the initial distribution of balls?

(B4) Find all pairs of polynomials p(x) and q(x) with real coefficients for which

p(x)q(x + 1) − p(x + 1)q(x) = 1.

Page 16
Putnam In Olympiad Mahdiyar Amid Sardari

2009
(A1) Let f be a real-valued function on the plane such that for every square
ABCD in the plane, f (A) + f (B) + f (C) + f (D) = 0. Does it follow that
f (P ) = 0 for all points P in the plane?

(A4) Let S be a set of rational numbers such that


(1) 0 ∈ S;
(2) If x ∈ S then x + 1 ∈ S and x − 1 ∈ S; and
1
(3) If x ∈ S and x ∈
/ {0, 1}, then x(x−1) ∈ S.
Must S contain all rational numbers?

(A5) Is there a finite Abelian group G such that the product of the orders
of all its elements is 22009 ?

(B1) Show that every positive rational number can be written as a quotient
of products of factorials of (not necessarily distinct) primes.

(B2) A game involves jumping to the right on the real number line. If a and
b are real numbers and b > a, the cost of jumping from a to b is b3 − ab2 . For
what real numbers c can one travel from 0 to 1 in a finite number of jumps with
total cost exactly c?

(B3) Call a subset S of {1, 2, . . . , n} mediocre if it has the following property:


Whenever a and b are elements of S whose average is an integer, that aver-
age is also an element of S. Let A(n) be the number of mediocre subsets of
{1, 2, . . . , n}. [For instance, every subset of {1, 2, 3} except {1, 3} is mediocre, so
A(3) = 7.] Find all positive integers n such that A(n+2)−2A(n+1)+A(n) = 1.

(B4) Say that a polynomial with real coefficients in two variable, x, y, is bal-
anced if the average value of the polynomial on each circle centered at the origin
is 0. The balanced polynomials of degree at most 2009 form a vector space V
over R. Find the dimension of V.

(B6) Prove that for every positive integer n, there is a sequence of integers
a0 , a1 , . . . , a2009 with a0 = 0 and a2009 = n such that each term after a0 is
either an earlier term plus 2k for some non negative integer k, or of the form b
mod c for some earlier positive terms b and c.

2008
(A1) Let f : R2 → R be a function such that f (x, y) + f (y, z) + f (z, x) = 0 for
real numbers x, y, and z. Prove that there exists a function g : R → R such that

Page 17
Putnam In Olympiad Mahdiyar Amid Sardari

f (x, y) = g(x) − g(y) for all real numbers x and y.

(A2) Alan and Barbara play a game in which they take turns filling entries
of an initially empty 2008 × 2008 array. Alan plays first. At each turn, a player
chooses a real number and places it in a vacant entry. The game ends when
all entries are filled. Alan wins if the determinant of the resulting matrix is
nonzero; Barbara wins if it is zero. Which player has a winning strategy?

(A3) Start with a finite sequence a1 , a2 , . . . , an of positive integers. If possi-


ble, choose two indices j < k such that aj does not divide ak and replace aj
and ak by gcd(aj , ak ) and lcm (aj , ak ), respectively. Prove that if this process
is repeated, it must eventually stop and the final sequence does not depend on
the choices made.

(A5) Let n ≥ 3 be an integer. Let f (x) and g(x) be polynomials with real
coefficients such that the points (f (1), g(1)), (f (2), g(2)), . . . , (f (n), g(n)) in R2
are the vertices of a regular n-gon in counterclockwise order. Prove that at least
one of f (x) and g(x) has degree greater than or equal to n − 1.

(B1) What is the maximum number of rational points that can lie on a cir-
cle in R2 whose center is not a rational point? (A rational point is a point both
of whose coordinates are rational numbers.)

(B3) What is the largest possible radius of a circle contained in a 4-dimensional


hypercube of side length 1?

(B4) Let p be a prime number. Let h(x) be a polynomial with integer coef-
ficients such that h(0), h(1), . . . , h(p2 − 1) are distinct modulo p2 . Show that
h(0), h(1), . . . , h(p3 − 1) are distinct modulo p3 .

(B5) Find all continuously differentiable functions f : R → R such that for


every rational number q, the number f (q) is rational and has the same denom-
inator as q. (The denominator of a rational number q is the unique positive
integer b such that q = a/b for some integer a with gcd(a, b) = 1.)

(B6) Let n and k be positive integers. Say that a permutation σ of {1, 2, . . . n}


is k-limited if |σ(i) − i| ≤ k for all i. Prove that the number of k-limited permu-
tations of {1, 2, . . . n} is odd if and only if n ≡ 0 or 1 (mod 2k + 1).

2007
1
(A1) Find all values of α for which the curves y = αx2 + αx + 24 and x =
1
αy 2 + αy + 24 are tangent to each other.

Page 18
Putnam In Olympiad Mahdiyar Amid Sardari

(A2) Find the least possible area of a convex set in the plane that intersects
both branches of the hyperbola xy = 1 and both branches of the hyperbola
xy = −1. (A set S in the plane is called convex if for any two points in S the
line segment connecting them is contained in S.)

(A3) Let k be a positive integer. Suppose that the integers 1, 2, 3, . . . , 3k + 1 are


written down in random order. What is the probability that at no time during
this process, the sum of the integers that have been written up to that time is a
positive integer divisible by 3? Your answer should be in closed form, but may
include factorials.

(A4) A repunit is a positive integer whose digits in base 10 are all ones. Find
all polynomials f with real coefficients such that if n is a repunit, then so is f (n).

(A5) Suppose that a finite group has exactly n elements of order p, where p
is a prime. Prove that either n = 0 or p divides n + 1.

(A6) A triangulation T of a polygon P is a finite collection of triangles whose


union is P, and such that the intersection of any two triangles is either empty,
or a shared vertex, or a shared side. Moreover, each side of P is a side of exactly
one triangle in T . Say that T is admissible if every internal vertex is shared by
6 or more triangles. Prove that there is an integer Mn , depending only on n,
such that any admissible triangulation of a polygon P with n sides has at most
Mn triangles.

(B1) Let f be a polynomial with positive integer coefficients. Prove that if


n is a positive integer, then f (n) divides f (f (n) + 1) if and only if n = 1.
 √ 
(B3) Let x0 = 1 and for n ≥ 0, let xn+1 = 3xn + xn 5 . In particular,
x1 = 5, x2 = 26, x3 = 136, x4 = 712. Find a closed-form expression for x2007 .
(⌊a⌋ means the largest integer ≤ a.)

(B4) Let n be a positive integer. Find the number of pairs P, Q of polyno-


mials with real coefficients such that
(P (X))2 + (Q(X))2 = X 2n + 1
and degP < degQ.

(B5) Let k be a positive integer. Prove that there exist polynomials P0 (n), P1 (n), . . . , Pk−1 (n)
(which may depend on k) such that for any integer n,
j n kk jnk j n kk−1
= P0 (n) + P1 (n) + · · · + Pk−1 (n) .
k k k
(⌊a⌋ means the largest integer ≤ a.)

Page 19
Putnam In Olympiad Mahdiyar Amid Sardari

(B6) For each positive integer n, let f (n) be the number of ways to make
n! cents using an unordered collection of coins, each worth k! cents for some
k, 1 ≤ k ≤ n. Prove that for some constant C, independent of n,
2
/2−Cn −n2 /4 2
/2+Cn −n2 /4
nn e ≤ f (n) ≤ nn e .

2006
(A1) Find the volume of the region of points (x, y, z) such that
2
x2 + y 2 + z 2 + 8 ≤ 36 x2 + y 2 .


(A2) Alice and Bob play a game in which they take turns removing stones
from a heap that initially has n stones. The number of stones removed at each
turn must be one less than a prime number. The winner is the player who takes
the last stone. Alice plays first. Prove that there are infinitely many such n
such that Bob has a winning strategy.

(A3) Let 1, 2, 3, . . . , 2005, 2006, 2007, 2009, 2012, 2016, . . . be a sequence defined
by xk = k for k = 1, 2 . . . , 2006 and xk+1 = xk + xk−2005 for k ≥ 2006. Show
that the sequence has 2005 consecutive terms each divisible by 2006.

(A4) Let S = {1, 2 . . . , n} for some integer n > 1. Say a permutation π of


S has a local maximum at k ∈ S if
(1) π(k) > π(k + 1) for k = 1
(2) π(k − 1) < π(k) and π(k) > π(k + 1) for 1 < k < n
(3) π(k − 1)M π(k) for k = n

What is the average number of local maxima of a permutation of S, averaging


over all permutations of S?

(A5) Let n be a positive odd integer and let θ be a real number such that
θ/π is irrational. Set ak = tan(θ + kπ/n), k = 1, 2 . . . , n. Prove that
a1 + a2 + · · · + an
a1 a2 · · · an
is an integer, and determine its value.

(A6) Four points are chosen uniformly and independently at random in the
interior of a given circle. Find the probability that they are the vertices of a

Page 20
Putnam In Olympiad Mahdiyar Amid Sardari

convex quadrilateral.

(B1) Show that the curve x3 + 3xy + y 3 = 1 contains only one set of three
distinct points, A, B, and C, which are the vertices of an equilateral triangle.

(B2) Prove that, for every set X = {x1 , x2 , . . . , xn } of n real numbers, there
exists a non-empty subset S of X and an integer m such that

X 1
m + s ≤

n+1
s∈S

(B3) Let S be a finite set of points in the plane. A linear partition of S is


an unordered pair {A, B} of subsets of S such that A ∪ B = S, A ∩ B = ∅, and
A and B lie on opposite sides of some straight line disjoint from S (A or B may
be empty). Let LS be the number of linear partitions of S. For each positive
integer n, find the maximum of LS over all sets S of n points.

(B4) Let Z denote the set of points in Rn whose coordinates are 0 or 1. (Thus
Z has 2n elements, which are the vertices of a unit hypercube in Rn .) Given a
vector subspace V of Rn , let Z(V ) denote the number of members of Z that lie
in V. Let k be given, 0 ≤ k ≤ n. Find the maximum, over all vector sub spaces
V ⊆ Rn of dimension k, of the number of points in V ∩ Z.

(B6) Let k be an integer greater than 1. Suppose a0 > 0 and define


1
an+1 = an + √
k a
n

for n ≥ 0. Evaluate
ak+1
n
lim .
n→∞ nk

2005
(A1) Show that every positive integer is a sum of one or more numbers of the
form 2r 3s , where r and s are non negative integers and no summoned divides
another.

(A2) Let S = {(a, b)|a = 1, 2, . . . , n, b = 1, 2, 3}. A rook tour of S is a polygonal


path made up of line segments connecting points p1 , p2 , . . . , p3n is sequence such
that
(1) pi ∈ S,

Page 21
Putnam In Olympiad Mahdiyar Amid Sardari

(2) pi and pi+1 are a unit distance apart, for 1 ≤ i < 3n,
(3) for each p ∈ S there is a unique i such that pi = p.
How many rook tours are there that begin at (1, 1) and end at (n, 1)?

(A3) Let p(z) be a polynomial of degree n, all of whose zeros have absolute
value 1 in the complex plane. Put g(z) = zp(z) ′
n/2 . Show that all zeros of g (z) = 0

have absolute value 1.

(A6) Let n be given, n ≥ 4, and suppose that P1 , P2 , . . . , Pn are n randomly,


independently and uniformly, chosen points on a circle. Consider the convex
n-gon whose vertices are the Pi . What is the probability that at least one of the
vertex angles of this polygon is acute.?

(B1) Find a nonzero polynomial P (x, y) such that P (⌊a⌋, ⌊2a⌋) = 0 for all
real numbers a.
(Note: ⌊v⌋ is the greatest integer less than or equal to v.)

(B2) Find all positive integers n, k1 , . . . , kn such that k1 + · · · + kn = 5n − 4 and


1 1
+ ··· + = 1.
k1 kn

(B4) For positive integers m and n, let f (m, n) denote the number of n-tuples
(x1 , x2 , . . . , xn ) of integers such that |x1 | + |x2 | + · · · + |xn | ≤ m. Show that
f (m, n) = f (n, m).

(B5) Let P (x1 , . . . , xn ) denote a polynomial with real coefficients in the vari-
ables
 x21 , . . . , xn , and suppose that
∂ ∂2
(1) ∂x2 + · · · + ∂x2 P (x1 , . . . , xn ) = 0 (identically)
1 n
and that
(2) x21 + · · · + x2n divides P (x1 , . . . , xn ).
Show that P = 0 identically.

(B6) Let Sn denote the set of all permutations of the numbers 1, 2, . . . , n. For
π ∈ Sn , let σ(π) = 1 if π is an even permutation and σ(π) = −1 if π is an odd
permutation. Also, let v(π) denote the number of fixed points of π. Show that
X σ(π) n
= (−1)n+1 .
v(π) + 1 n+1
π∈Sn

Page 22
Putnam In Olympiad Mahdiyar Amid Sardari

2004
(A1) Basketball star Shanille O’Keal’s team statistician keeps track of the num-
ber, S(N ), of successful free throws she has made in her first N attempts of the
season. Early in the season, S(N ) was less than 80

(A2) For i = 1, 2, let Ti be a triangle with side length ai , bi , ci , and area Ai .


Suppose that a1 ≤ a2 , b1 ≤ b2 , c1 ≤ c2 , and that T2 is an acute triangle. Does
it follow that A1 ≤ A2 ?

(A4) Show that for any positive integer n there is an integer N such that the
product x1 x2 · · · xn can be expressed identically in the form
N
X
x1 x2 · · · xn = ci (ai1 x1 + ai2 x2 + · · · + ain xn )n
i=1

where the ci are rational numbers and each aij is one of the numbers, −1, 0, 1.

(A5) An m × n checkerboard is colored randomly: each square is independently


assigned red or black with probability 21 . we say that two squares, p and q, are
in the same connected monochromatic region if there is a sequence of squares,
all of the same color, starting at p and ending at q, in which successive squares
in the sequence share a common side. Show that the expected number of con-
nected monochromatic regions is greater than mn 8 .

(B1) Let P (x) = cn xn + cn−1 xn−1 + · · · + c0 be a polynomial with integer coeffi-


cients. Suppose that r is a rational number such that P (r) = 0. Show that the n
numbers cn r, cn r2 +cn−1 r, cn r3 +cn−1 r2 +cn−1 r, . . . , cn rn +cn−1 rn−1 +· · ·+c1 r
are all integers.
(m+n)! m! n!
(B2) Let m and n be positive integers. Show that (m+n)m+n < mm · nn

(B3) Determine all real numbers a > 0 for which there exists a non nega-
tive continuous function f (x) defined on [0, a] with the property that the region
R = {(x, y) : 0 ≤ x ≤ a, 0 ≤ y ≤ f (x)} has perimeter k units and area k square
units for some real number k.

(B4) Let n be a positive integer, n ≥ 2, and put θ = 2π n . Define points


Pk = (k, 0) in the xy-plane, for k = 1, 2, . . . , n. Let Rk be the map that rotates
the plane counterclockwise by the angle θ about the point Pk . Let R denote the
map obtained by applying in order, R1 , then R2 , ..., then Rn . For an arbitrary
point (x, y), find and simplify the coordinates of R(x, y).

Page 23
Putnam In Olympiad Mahdiyar Amid Sardari

2003
(A1) Let n be a fixed positive integer. How many ways are there to write n as
a sum of positive integers,

n = a1 + a2 + · · · ak

with k an arbitrary positive integer and a1 ≤ a2 ≤ · · · ≤ ak ≤ a1 + 1?

(A2) Let a1 , a2 , · · · , an and b1 , b2 , · · · , bn be non negative real numbers. Show


that

(a1 a2 · · · an )1/n + (b1 b2 · · · bn )1/n ≤ ((a1 + b1 )(a2 + b2 ) · · · (an + bn ))1/n

(A3) Find the minimum value of

| sin x + cos x + tan x + cot x + sec x + csc x|

for real numbers x.

(A4) Suppose that a, b, c, A, B, C are real numbers, a ̸= 0 and A ̸= 0, such


that
|ax2 + bx + c| ≤ |Ax2 + Bx + C|
for all real numbers x. Show that

|b2 − 4ac| ≤ |B 2 − 4AC|

(A5) A Dyck n-path is a lattice path of n upsteps (1, 1) and n downsteps


(1, −1) that starts at the origin O and never dips below the x-axis. A return
is a maximal sequence of contiguous downsteps that terminates on the x-axis.
For example, the Dyck 5-path illustrated has two returns, of length 3 and 1
respectively. Show that there is a one-to-one correspondence between the Dyck
n-paths with no return of even length and the Dyck (n − 1) paths.

àbc
bcà àbc @ bcà
@
àbc @@àbc @àbc àbc
àbc @ @àbc @@àbc
O

(A6) For a set S of non negative integers, let rS (n) denote the number of

Page 24
Putnam In Olympiad Mahdiyar Amid Sardari

ordered pairs (s1 , s2 ) such that s1 ∈ S, s2 ∈ S, s1 ̸= s2 , and s1 + s2 = n. Is it


possible to partition the non negative integers into two sets A and B in such a
way that rA (n) = rB (n) for all n?

(B1) Do there exist polynomials a(x), b(x), c(y), d(y) such that

1 + xy + x2 y 2 = a(x)c(y) + b(x)d(y)

holds identically?

(B2) Let n be a positive integer. Starting with the sequence 1, 21 , 31 , · · · , n1 ,


form a new sequence of n − 1 entries 34 , 125 2n−1
, · · · , 2n(n−1) , by taking the aver-
ages of two consecutive entries in the first sequence. Repeat the averaging of
neighbors on the second sequence to obtain a third sequence of n − 2 entries
and continue until the final sequence consists of a single number xn . Show that
xn < n2 .

(B3) Show that for each positive integer n,


n
Y jnk
n! = lcm {1, 2, . . . , }
i=1
i

(Here lcm denotes the least common multiple, and ⌊x⌋ denotes the greatest in-
teger ≤ x.)

(B4) Let f (z) = az 4 + bz 3 + cz 2 + dz + e = a(z − r1 )(z − r2 )(z − r3 )(z − r4 )


where a, b, c, d, e are integers, a ̸= 0. Show that if r1 + r2 is a rational number,
and if r1 + r2 ̸= r3 + r4 , then r1 r2 is a rational number.

(B5) Let A, B and C be equidistant points on the circumference of a circle


of unit radius centered at O, and let P be any point in the circle’s interior. Let
a, b, c be the distances from P to A, B, C respectively. Show that there is a
triangle with side lengths a, b, c, and that the area of this triangle depends only
on the distance from P to O.

2002
1
(A1) Let k be a fixed positive integer. The nth derivative of xk −1
has the form
Pn (x)
(xk −1)n+1
, where Pn (x) is a polynomial. Find Pn (1).

(A2) Given any five points on a sphere, show that some four of them must
lie on a closed hemisphere.

Page 25
Putnam In Olympiad Mahdiyar Amid Sardari

(A3) Let N be an integer greater than 1 and let Tn be the number of non
empty subsets S of {1, 2, ....., n} with the property that the average of the ele-
ments of S is an integer. Prove that Tn − n is always even.

(A5) Define a sequence by a0 = 1, together with the rules a2n+1 = an and


a2n+2 = an + an+1 for eachn integer n ≥o0. Prove that every positive rational
an−1 1 1 2 1 3
number appears in the set an : n ≥ 1 = 1 , 2 , 1 , 3 , 2 , · · · .

(B1) Shanille O’Keal shoots free throws on a basketball court. She hits the
first and misses the second, and thereafter the probability that she hits the next
shot is equal to the proportion of shots she has hit so far. What is the proba-
bility she hits exactly 50 of her first 100 shots?

(B2) Consider a polyhedron with at least five faces such that exactly three
edges emerge from each of its vertices. Two players play the following game:
Each, in turn, signs his or her name on a previously unsigned face. The winner
is the player who first succeeds in signing three faces that share a common ver-
tex. Show that the player who signs first will always win by playing as well as
possible.

(B4) An integer n, unknown to you, has been randomly chosen in the inter-
val [1, 2002] with uniform probability. Your objective is to select n in an odd
number of guess. After each incorrect guess, you are informed whether n is
higher or lower, and you must guess an integer on your next turn among the
numbers that are still feasibly correct. Show that you have a strategy so that
the chance of winning is greater than 23 .

(B5) A palindrome in base b is a positive integer whose base-b digits read the
same backwards and forwards; for example, 2002 is a 4-digit palindrome in base
10. Note that 200 is not a palindrome in base 10, but it is a 3-digit palindrome:
242 in base 9, and 404 in base 7. Prove that there is an integer which is a 3-digit
palindrome in base b for at least 2002 different values of b.

2001
1
(A2) For each k, Ck is biased so that, when tossed, it has probability (2k+1) of
falling heads. If the n coins are tossed, what is the probability that the number
of heads is odd? Express the answer as a rational function n.

(A3) For each integer m, consider the polynomial

Pm (x) = x4 − (2m + 4)x2 + (m − 2)2 .

Page 26
Putnam In Olympiad Mahdiyar Amid Sardari

For what values of m is Pm (x) the product of two non-constant polynomials


with integer coefficients?

(A4) Triangle ABC has area 1. Points E, F , and G lie, respectively, on sides
BC, CA, and AB such that AE bisects BF at point R, BF bisects CG at point
S, and CG bisects AE at point T . Find the area of the triangle RST .

(A5) Prove that there are unique positive integers a, n such that an+1 −(a+1)n =
2001.

(B1) Let n be an even positive integer. Write the numbers 1, 2, · · · , n2 in the


squares of an n × n grid so that the kth row, from left to right, is

(k − 1)n + 1, (k − 1)n + 2, · · · , (k − 1)n + n.

Color the squares of the grid so that half of the squares in each row and in
each column are red and the other half are black (a checkerboard coloring is one
possibility). Prove that for each coloring, the sum of the numbers on the red
squares is equal to the sum of the numbers on the black squares.

(B2) Find all pairs of real numbers (x, y) satisfying the system of equations:
1 1
+ = (x2 + 3y 2 )(3x2 + y 2 )
x 2y
1 1
− = 2(y 4 − x4 )
x 2y


(B3) For any positive integer n, let ⟨n⟩ denote the closest integer to n. Eval-
uate:

X 2⟨n⟩ + 2−⟨n⟩
n=1
2n

(B4) Let S denote the set of rational numbers different from {−1, 0, 1}. De-
fine f : S → S by f (x) = x − 1/x. Prove or disprove that

∩∞
n=1 f
(n)
(S) = ∅

where f (n) denotes f composed with itself n times.

(B5) Let a and b be real numbers in the interval 0, 12 , and let g be a con-


tinuous real-valued function such that g(g(x)) = ag(x) + bx for all real x. Prove
that g(x) = cx for some constant c.

(B6) Assume that (an )n≥1 is an increasing sequence of positive real numbers

Page 27
Putnam In Olympiad Mahdiyar Amid Sardari

such that lim ann = 0. Must there exist infinitely many positive integers n such
that an−i + an+i < 2an for i = 1, 2, · · · , n − 1?

2000

X
(A1) Let A be a positive real number. What are the possible values of x2j ,
j=0

X
given that x0 , x1 , · · · are positive numbers for which xj = A?
j=0

(A2) Prove that there exist infinitely many integers n such that n, n + 1, n + 2
are each the sum of the squares of two integers.

(A3) The octagon P1 P2 P3 P4 P5 P6 P7 P8 is inscribed in a circle with the vertices


around the circumference in the given order. Given that the polygon P1 P3 P5 P7
is a square of area 5, and the polygon P2 P4 P6 P8 is a rectangle of area 4, find
the maximum possible area of the octagon.

(A5) Three distinct points with integer coordinates lie in the plane on a cir-
cle of radius r > 0. Show that two of these points are separated by a distance
of at least r1/3 .

(A6) Let f (x) be a polynomial with integer coefficients. Define a sequence


a0 , a1 , · · · of integers such that a0 = 0 and an+1 = f (an ) for all n ≥ 0. Prove
that if there exists a positive integer m for which am = 0 then either a1 = 0 or
a2 = 0.

(B1) Let aj , bj , cj be integers for 1 ≤ j ≤ N . Assume for each j, at least one of


aj , bj , cj is odd. Show that there exists integers r, s, t such that raj + sbj + tcj
is odd for at least 4N 7 values of j, 1 ≤ j ≤ N .

(B2) Prove that the expression


 
gcd(m, n) n
n m

is an integer for all pairs of integers n ≥ m ≥ 1.

(B4) Let f (x) be a continuous function such that f (2x2 − 1) = 2xf (x) for
all x. Show that f (x) = 0 for −1 ≤ x ≤ 1.

(B5) Let S0 be a finite set of positive integers. We define finite sets S1 , S2 , · · ·


of positive integers as follows: the integer a in Sn+1 if and only if exactly one of

Page 28
Putnam In Olympiad Mahdiyar Amid Sardari

a − 1 or a is in Sn . Show that there exist infinitely many integers N for which


SN = S0 ∪ {N + a : a ∈ S0 }.
n+1
(B6) Let B be a set of more than 2 n distinct points with coordinates of
the form (±1, ±1, · · · , ±1) in n-dimensional space with n ≥ 3. Show that there
are three distinct points in B which are the vertices of an equilateral triangle.

1999
(A1) Find polynomials f (x), g(x), and h(x), if they exist, such that for all x,
|f (x)| − |g(x)| + h(x) = −1 if x < −1
|f (x)| − |g(x)| + h(x) = 3x + 2 if − 1 ≤ x ≤ 0
|f (x)| − |g(x)| + h(x) = −2x + 2 if x > 0

(A2) Let p(x) be a polynomial that is non negative for all real x. Prove that
for some k, there are polynomials f1 (x), f2 (x), . . . , fk (x) such that
k
X
p(x) = (fj (x))2 .
j=1

(A3) Consider the power series expansion



1 X
= an xn .
1 − 2x − x2 n=0

Prove that, for each integer n ≥ 0, there is an integer m such that


a2n + a2n+1 = am .

(A4) Sum the series


∞ X

X m2 n
.
m=1 n=1
3m (n3m + m3n )

(A6) The sequence (an )n≥1 is defined by a1 = 1, a2 = 2, a3 = 24, and, for


n ≥ 4,
6a2 an−3 − 8an−1 a2n−2
an = n−1 .
an−2 an−3

Page 29
Putnam In Olympiad Mahdiyar Amid Sardari

Show that, for all n, an is an integer multiple of n.

(B1) Right triangle ABC has right angle at C and ̸ BAC = θ; the point
D is chosen on AB so that |AC| = |AD| = 1; the point E is chosen on BC so
that ̸ CDE = θ. The perpendicular to BC at E meets AB at F . Evaluate
limθ→0 |EF |.

(B2) Let P (x) be a polynomial of degree n such that P (x) = Q(x)P ′′ (x), where
Q(x) is a quadratic polynomial and P ′′ (x) is the second derivative of P (x).
Show that if P (x) has at least two distinct roots then it must have n distinct
roots.

(B3) Let A = {(x, y) : 0 ≤ x, y < 1}. For (x, y) ∈ A, let


X
S(x, y) = xm y n ,
1 m
2 ≤ n ≤2

where the sum ranges over all pairs (m, n) of positive integers satisfying the
indicated inequalities. Evaluate

lim (1 − xy 2 )(1 − x2 y)S(x, y).


(x,y)→(1,1),(x,y)∈A

(B6) Let S be a finite set of integers, each greater than 1. Suppose that for
each integer n there is some s ∈ S such that gcd(s, n) = 1 or gcd(s, n) = s.
Show that there exist s, t ∈ S such that gcd(s, t) is prime.

1998
(A1) A right circular cone has base of radius 1 and height 3. A cube is inscribed
in the cone so that one face of the cube is contained in the base of the cone.
What is the side-length of the cube?

(A2) Let s be any arc of the unit circle lying entirely in the first quadrant.
Let A be the area of the region lying below s and above the x-axis and let
B be the area of the region lying to the right of the y-axis and to the left of
s. Prove that A+B depends only on the arc length, and not on the position, of s.

(A4) Let A1 = 0 and A2 = 1. For n > 2, the number An is defined by


concatenating the decimal expansions of An−1 and An−2 from left to right. For
example A3 = A2 A1 = 10, A4 = A3 A2 = 101, A5 = A4 A3 = 10110, and so
forth. Determine all n such that 11 divides An .

Page 30
Putnam In Olympiad Mahdiyar Amid Sardari

(A6) Let A, B, C denote distinct points with integer coefficients in R2 . Prove


that if
(|AB| + |BC|)2 < 8 · [ABC] + 1
then A, B, C are three vertices of a square. Here |XY | is the length of segment
XY and [ABC] is the area of triangle ABC.

(B1) Find the minimum value of

(x + 1/x)6 − (x6 + 1/x6 ) − 2


(x + 1/x)3 + (x3 + 1/x3 )

for x > 0.

(B2) Given a point (a, b) with 0 < b < a, determine the minimum perime-
ter of a triangle with one vertex at (a, b), one on the x-axis, and one on the line
y = x. You may assume that a triangle of minimum perimeter exists.

(B3) Let H be the unit hemisphere {(x, y, z) : x2 + y 2 + z 2 = 1, z ≥ 0}, C


the unit circle {(x, y, 0) : x2 + y 2 = 1}, and P the regular pentagon inscribed in
C. Determine the surface area of that portion of H lying over the planar region
inside P , and write your answer in the form A sin α + B cos β, where A, B, α, β
are real numbers.

(B4) Find necessary and sufficient conditions on positive integers m and n so


that
mn−1
X
(−1)⌊i/m⌋+⌊i/n⌋ = 0.
i=0

(B5) Let N be the positive integer with 1998 decimal digits, all of them 1;
that is,
N = 1111 · · · 11.

Find the thousandth digit after the decimal point of N .

(B6) √
Prove that, for any integers a, b, c, there exists a positive integer n such
that n3 + an2 + bn + c is not an integer.

1997
(A1) A rectangle, HOM F , has sides HO = 11 and OM = 5. A triangle ∆ABC
has H as orthocentre, O as circumcentre, M be the midpoint of BC, F is the
feet of altitude from A. What is the length of BC?

Page 31
Putnam In Olympiad Mahdiyar Amid Sardari

(A2) Players 1, 2, . . . n are seated around a table, and each has a single penny.
Player 1 passes a penny to Player 2, who then passes two pennies to Player 3,
who then passes one penny to player 4, who then passes two pennies to Player
5 and so on, players alternately pass one or two pennies to the next player who
still has some pennies. The player who runs out of pennies drops out of the
game and leaves the table. Find an infinite set of numbers n for which some
player ends up with all the n pennies.

(A4) Let Nk denote number of ordered n-tuples of positive integers (a1 , a2 , · · · , ak )


such that
1 1 1
+ + ... + =1
a1 a2 ak
Determine N10 is odd or even.

(A6) For a positive integer n and any real number c, define xk recursively by :
cxk+1 − (n − k)xk
x0 = 0, x1 = 1 and for k ≥ 0, xk+2 =
k+1
Fix n and then take c to be the largest value for which xn+1 = 0. Find xk in
terms of n and k, 1 ≤ k ≤ n.

(B1) For all reals x define {x} to be the difference between x and the clos-
est integer to x. For each positive integer n evaluate :
6n−1
X n m o n m o
Sn = min ,
m=1
6n 3n

Pn 1 pn
(B3) For each positive integer n write the sum i= i = qn with gcd(pn , qn ) = 1.
Find all such n such that 5qn .

(B4) Let am,n denote the coefficient of xn in the expansion (1 + x + x2 )n .


Prove the inequality for all integers k ≥ 0 :
⌊X3 ⌋
2k

0≤ (−1)ℓ ak−ℓ,ℓ ≤ 1
ℓ=0

(B5) Let us define a sequence {an }n≥1 . Define as follows:


a1 = 2 and an+1 = 2an for n ≥ 1
Show this :
an ≡ an−1 (mod n)

Page 32
Putnam In Olympiad Mahdiyar Amid Sardari

1996
(A1) Find the least number A such that for any two squares of combined area
1, a rectangle of area A exists such that the two squares can be packed in the
rectangle (without the interiors of the squares overlapping) . You may assume
the sides of the squares will be parallel to the sides of the rectangle.

(A2) Let C1 and C2 be circles whose centers are 10 units apart, and whose
radii are 1 and 3. Find, with proof, the locus of all points M for which there
exists points X ∈ C1 , Y ∈ C2 such that M is the midpoint of XY .

(A3) Suppose that each of 20 students has made a choice of anywhere from
0 to 6 courses from a total of 6 courses offered. Prove or disprove : there are
5 students and 2 courses such that all 5 have chosen both courses or all 5 have
chosen neither course.

(A4) S be a set of ordered triples (a, b, c) of distinct elements of a finite set


A. Suppose that
(1) (a, b, c) ∈ S ⇐⇒ (b, c, a) ∈ S
(2) (a, b, c) ∈ S ⇐⇒ (c, b, a) ̸∈ S
(3) (a, b, c), (c, d, a) both ∈ S ⇐⇒ (b, c, d), (d, a, b) both ∈ S
Prove there exists g : A → R, such that g is one-one and g(a) < g(b) < g(c) =⇒
(a, b, c) ∈ S

(A5) Let p be a prime greater than 3. Prove that

⌊X 3 ⌋ 
2p

2 p
p .
i=1
i

(A6) Let c ≥ 0 be a real number. Give a complete description with proof


of the set of all continuous functions f : R → R such that f (x) = f (x2 + c) for
all x ∈ R.

(B1) Define a selfish set to be a set which has its own cardinally as its ele-
ment. And a set is a minimal selfish set if none of its proper subsets are selfish.
Find with proof the number of minimal selfish subsets of {1, 2, · · · , n}.

(B3) Let Sn be the set of all permutations of (1, 2, . . . , n). Then find :
n
!
X
max σ(i)σ(i + 1)
σ∈Sn
i=1

where σ(n + 1) = σ(1).

Page 33
Putnam In Olympiad Mahdiyar Amid Sardari

(B5) Given a finite binary string S of symbols X, O we define ∆(S) = n(X) −


n(O) where n(X), n(O) respectively denote number of X’s and O’s in a string.
For example ∆(XOOXOOX) = 3 − 4 = −1. We call a string S balanced if
every sub string T of S has −2 ≤ ∆(T ) ≤ 2. Find number of balanced strings
of length n.

(B6) Let (a1 , b1 ), (a2 , b2 ), . . . , (an , bn ) be the vertices of a convex polygon con-
taining the origin in its interior. Prove that there are positive real numbers x, y
such that :

(a1 , b1 )xa1 y b1 + (a2 , b2 )xa2 y b2 + . . . + (an , bn )xan y bn = (0, 0)

1995
(A1) Let S be a set of real numbers which is closed under multiplication (that
is a, b ∈ S =⇒ ab ∈ S). Let T, U ⊂ S such that T ∩ U = ∅, T ∪ U = S. Given
that for any three elements a, b, c in T , not necessarily distinct, we have abc ∈ T
and also if a, b, c ∈ U , not necessarily distinct then abc ∈ U . Show at least one
of T and U is closed under multiplication.

(A3) The number d1 d2 · · · d9 has nine (not necessarily distinct) decimal dig-
its. The number e1 e2 · · · e9 is such that each of the nine 9-digit numbers formed
by replacing just one of the digits di in d1 d2 · · · d9 by the corresponding digit
ei (1 ≤ i ≤ 9) is divisible by 7. The number f1 f2 · · · f9 is related to e1 e2 · · · e9
is the same way: that is, each of the nine numbers formed by replacing one of
the ei by the corresponding fi is divisible by 7. Show that, for each i, di − fi is
divisible by 7.

(A4) Suppose we have a necklace of n beads. Each bead is labelled with an


integer and the sum of all these labels is n − 1. Prove that we can cut the
necklace to form a string whose consecutive labels x1 , x2 , · · · , xn satisfy
k
X
xi ≤ k − 1 ∀ 1≤k≤n
i=1

(A6) Suppose that each of n people writes down the numbers 1, 2, 3 in random
order in one column of a 3× n matrix, with all orders equally likely and with the
orders for different columns independent of each other. Let the row sums a, b, c
of the resulting matrix be rearranged (if necessary) so that a ≤ b ≤ c. Show
that for some n ≥ 1995 ,it is at least four times as likely that both b = a + 1
and c = a + 2 as that a = b = c.

Page 34
Putnam In Olympiad Mahdiyar Amid Sardari

(B1) For a partition π of {1, 2, 3, 4, 5, 6, 7, 8, 9}, let π(x) be the number of ele-
ments in the part containing x. Prove that for any two partitions π and π ′ , there
are two distinct numbers x and y in {1, 2, 3, 4, 5, 6, 7, 8, 9} such that π(x) = π(y)
and π ′ (x) = π ′ (y).

(B4) Evaluate : s
1
8 2207 − 1
2207 − 2207−···

a+b c
Express your expression in the form d where a, b, c, d ∈ Z.

A game starts with four heaps of beans, containing 3, 4, 5 and 6 beans. The
two players move alternately. A move consists of taking:
(1) either one bean from a heap, provided at least two beans are left behind in
that heap,
(2) or a complete heap of two or three beans. The player who takes the last
heap wins. To win the game, do you want to move first or second?
Give a winning strategy.

(B6) For any a > 0,set S(a) = {⌊na⌋|n ∈ N }. Show that there are no three
positive reals a, b, c such that

S(a) ∩ S(b) = S(b) ∩ S(c) = S(c) ∩ S(a) = ∅

S(a) ∪ S(b) ∪ S(c) = N

1994
(A1) Suppose that a sequence {an }n≥1 satisfies 0 < an ≤ a2n + a2n+1 for all
P∞
n ∈ N . Prove that the series n=1 an diverges.

(A2) Let A be the area of the region in the first quadrant bounded by the
2
line y = x2 , the x-axis, and the ellipse x9 + y 2 = 1. Find the positive number m
such that A is equal to the area of the region in the first quadrant bounded by
2
the line y = mx, the y-axis, and the ellipse x9 + y 2 = 1.

(A3) Show that if the points of an isosceles right triangle of side length 1 are
each colored with one of four colors, then
√ there must be two points of the same
color which are at least a distance 2 − 2 apart.

(A5) Let (rn )n≥0 be a sequence of positive real numbers such that limn→∞ rn =

Page 35
Putnam In Olympiad Mahdiyar Amid Sardari

0. Let S be the set of numbers re presentable as a sum

ri1 + ri2 + · · · + ri1994 ,

with i1 < i2 < · · · < i1994 . Show that every nonempty interval (a, b) contains a
nonempty sub interval (c, d) that does not intersect S.

(A6) Let f1 , f2 , · · · , f10 be bijections on Z such that for each integer n, there is
some composition fℓ1 ◦ fℓ2 ◦ · · · ◦ fℓm (allowing repetitions) which maps 0 to n.
Consider the set of 1024 functions

F = {f1ϵ1 ◦ f2ϵ2 ◦ · · · ◦ f10


ϵ10
}

where ϵi = 0 or 1 for 1 ≤ i ≤ 10. (fi0 is the identity function and fi1 = fi ). Show
that if A is a finite set of integers then at most 512 of the functions in F map
A into itself.

(B1) Find all positive integers that are within 250 of exactly 15 perfect squares.

(B2) For which real numbers c is there a straight line that intersects the curve

y = x4 + 9x3 + cx2 + 9x + 4

in four distinct points?

(B6) For a ∈ Z define


na = 101a − 100 · 2a
Show that, for 0 ≤ a, b, c, d ≤ 99

na + nb ≡ nc + nd (mod 10100) =⇒ {a, b} = {c, d}

1993
(A2) The sequence an of non-zero reals satisfies a2n − an−1 an+1 = 1 for n ≥ 1.
Prove that there exists a real number α such that an+1 = αan − an−1 for n ≥ 1.

(A3) Let P be the set of all subsets of 1, 2, ..., n. Show that there are 1n + 2n +
... + mn functions f : P 7−→ 1, 2, ..., m such that f (A ∩ B) = min(f (A), f (B))
for all A, B.

(A4) Given a sequence of 19 positive (not necessarily distinct) integers not


greater than 93, and a set of 93 positive (not necessarily distinct) integers not
greater than 19. Show that we can find non-empty sub sequences of the two

Page 36
Putnam In Olympiad Mahdiyar Amid Sardari

sequences with equal sum.

(A6) Let a0 , a1 , a2 , ... be a sequence such that: a0 = 2; each an = 2 or 3; an =the


number of 3s between the nth and n + 1th 2 in the sequence. So the sequence
starts: 233233323332332... . Show that we can find α such that an = 2 iff
n = [αm] for some integer m ≥ 0.

(B1) What is the smallest integer n > 0 such that for any integer m in the
range 1, 2, 3, ..., 1992 we can always find an integral multiple of n1 in the open
m
interval ( 1993 , m+1
1994 )?

(B2) A deck of 2n cards numbered from 1 to 2n is shuffled and n cards are


dealt to A and B. A and B alternately discard a card face up, starting with A.
The game when the sum of the discards is first divisible by 2n + 1, and the last
person to discard wins. What is the probability that A wins if neither player
makes a mistake?

(B5) Show that given any 4 points in the plane we can find two whose dis-
tance apart is not an odd integer.

(B6) Let S be a set of three, not necessarily distinct, positive integers. Show
that one can transform S into a set containing 0 by a finite number of applica-
tions of the following rule:
Select two of the integers x and y, where x ≤ y and replace them with 2x and
y − x.

1992
(A1) Find all functions f : Z → Z for which we have f (0) = 1 and f (f (n)) =
f (f (n + 2) + 2) = n, for every natural number n.

(A3) Let m, n are natural numbers such that gcd(m, n) = 1.Find all triplets
(x, y, n) which satisfy (x2 + y 2 )m = (xy)n

(A4) Let f be an infinitely differentiable real-valued function defined on the


2
real numbers. If f (1/n) = n2n+1 , n = 1, 2, 3, ..., Compute the values of the
derivatives of f k (0), k = 0, 1, 2, 3, ...

(A5) For each positive integer n, let an = 0 (or 1) if the number of 1’s in
the binary representation of n is even (or odd), respectively. Show that there
do not exist positive integers k and m such that

ak+j = ak+m+j = ak+2m+j

Page 37
Putnam In Olympiad Mahdiyar Amid Sardari

for 0 ≤ j ≤ m − 1.

(A6) Four points are chosen at random on the surface of a sphere. What is
the probability that the center of the sphere lies inside the tetrahedron whose
vertices are at the four points?

(B1) Let S be a set of n distinct real numbers. Let AS be the set of num-
bers that occur as averages of two distinct elements of S. For a given n ≥ 2,
what is the smallest possible number of elements in AS ?

(B2) For non negative integers n and k, define Q(n, k) to be the coefficient
of xk in the expansion (1 + x + x2 + x3 )n . Prove that
k   
X n n
Q(n, k) = .
j=0
j k − 2j

(B3) For any pair (x, y) of real numbers, a sequence (an (x, y)) is defined as
follows:
an (x, y)2 + y 2
a0 (x, y) = x, an+1 (x, y) = for n ≥ 0
2
Find the area of the region {(x, y) ∈ R2 | (an (x, y)) converges}.

1991
(A1) The rectangle with vertices (0, 0), (0, 3), (2, 0) and (2, 3) is rotated clock-
wise through a right angle about the point (2, 0), then about (5, 0), then about
(7, 0), and finally about (10, 0). The net effect is to translate it a distance 10
along the x-axis. The point initially at (1, 1) traces out a curve. Find the area
under this curve (in other words, the area of the region bounded by the curve,
the x-axis and the lines parallel to the y-axis through (1, 0) and (11, 0)).

(A3) Find all real polynomials p(x) of degree n ≥ 2 for which there exist real
numbers r1 < r2 < ... < rn such that
(2) p(ri ) = 0, 1≤ i ≤ n, and
ri +ri+1
(2) p′ 2 = 0, 1 ≤ i ≤ n − 1.

(A4) Can we find an (infinite) sequence of disks in the Euclidean plane such
that:
(1) their centers have no (finite) limit point in the plane;
(2) the total area of the disks is finite; and
(3) every line in the plane intersects at least one of the disks?

Page 38
Putnam In Olympiad Mahdiyar Amid Sardari

(A6) An n-sum of type 1 is a finite sequence of positive integers a1 , a2 , . . . , ar ,


such that:
(1) a1 + a2 + . . . + ar = n;
(2) a1 > a2 + a3 , a2 > a3 + a4 , . . . , ar−2 > ar−1 + ar , and ar−1 > ar .
For example, there are five 7-sums of type 1, namely: 7; 6, 1; 5, 2; 4, 3; 4, 2, 1.
An n-sum of type 2 is a finite sequence of positive integers b1 , b2 , . . . , bs such
that:
(1) b1 + b2 + . . . + bs = n;
(2) b1 ≥ b2 ≥ . . . ≥ bs ;
(3) each bi is in the sequence 1, 2, 4, . . . , gj , . . . defined by g1 = 1, g2 = 2,
gj = gj−1 + gj−2 + 1; and
(4) if b1 = gk , then 1, 2, 4, . . . , gk is a sub sequence.
For example, there are five 7-sums of type 2, namely: 4, 2, 1; 2, 2, 2, 1; 2, 2, 1, 1, 1;
2, 1, 1, 1, 1, 1; 1, 1, 1, 1, 1, 1, 1. Prove that for n ≥ 1 the number of type 1 and
type 2 n-sums is the same.

(B1) For each integer n ≥ 0, let S(n) = n − m2 , where m is the greatest


integer with m2 ≤ n. Define a sequence by a0 = A and ak+1 = ak + S(ak ) for
k ≥ 0. For what positive integers A is this sequence eventually constant?

(B2) Define functions f and g as non constant, differentiable, real-valued func-


tions on R. If f (x + y) = f (x)f (y) − g(x)g(y), g(x + y) = f (x)g(y) + g(x)f (y),
2 2
and f ′ (0) = 0, prove that (f (x)) + (g(x)) = 1 for all x.

(B3) Can we find N such that all m × n rectangles with m, n > N can be
tiled with 4 × 6 and 5 × 7 rectangles?
Pp p
 p+n

(B4) Let p > 2 be a prime. Prove that n=0 n n ≡ 2p + 1 (mod p2 ).

(B5) Let p > 2 be a prime. How many residues (mod p) are both squares
and squares plus one?

(B6) Let a and b be positive numbers. Find the largest number c, in terms
of a and b, such that for all x with 0 < |x| ≤ c and for all α with 0 < α < 1, we
have:
a sinh αx b sinh x(1 − α)
aα b1−α ≤ + .
sinh x sinh x

1990
(A1) Let
T0 = 2, T1 = 3, T2 = 6,

Page 39
Putnam In Olympiad Mahdiyar Amid Sardari

and for n ≥ 3,

Tn = (n + 4)Tn−1 − 4nTn−2 + (4n − 8)Tn−3 .

The first few terms are

2, 3, 6, 14, 40, 152, 784, 5158, 40576, 363392.

Find a formula for Tn of the form

Tn = An + Bn ,

where {An } and {Bn } are well known sequences.


√ √ √
(A2) Is 2 the limit of a sequence of numbers of the form 3 n − 3 m, where
n, m = 0, 1, 2, · · · .

(A3) Prove that any convex pentagon whose vertices (no three of which are
5
collinear) have integer coordinates must have area greater than or equal to .
2
(A4) Consider a paper punch that can be centered at any point of the plane
and that, when operated, removes from the plane precisely those points whose
distance from the center is irrational. How many punches are needed to remove
every point?

(A6) If X is a finite set, let X denote the number of elements in X. Call


an ordered pair (S, T ) of subsets of {1, 2, · · · , n} admissible if s > |T | for each
s ∈ S, and t > |S| for each t ∈ T . How many admissible ordered pairs of subsets
{1, 2, · · · , 10} are there? Prove your answer.

(B2) Prove that for |x| < 1, |z| > 1,



X
1 + xj Pj = 0,

1+
j=1

where Pj is
(1 − z)(1 − zx)(1 − zx2 ) · · · (1 − zxj−1 )
.
(z − x)(z − x2 )(z − x3 ) · · · (z − xj )

(B4) Let G be a finite group of order n generated by a and b. Prove or disprove:


there is a sequence
g1 , g2 , g3 , · · · , g2n
such that:
(1) every element of G occurs exactly twice, and
(2) gi+1 equals gi a or gi b for i = 1, 2, · · · , 2n. (interpret g2n+1 as g1 .)

Page 40
Putnam In Olympiad Mahdiyar Amid Sardari

(B5) Is there an infinite sequence a0 , a1 , a2 , · · · of nonzero real numbers such


that for n = 1, 2, 3, · · · the polynomial

pn (x) = a0 + a1 x + a2 x2 + · · · + an xn

has exactly n distinct real roots?

(B6) Let S be a nonempty closed bounded convex set in the plane. Let K
be a line and t a positive number. Let L1 and L2 be support lines for S parallel
to K1 , and let L be the line parallel to K and midway between L1 and L2 . Let
BS (K, t) be the band of points whose distance from L is at most 2t w, where
w is the distance between L1 and L2 . What is the smallest t such that
\
S∩ BS (K, t) ̸= ∅
K

for all S? (K runs over all lines in the plane.)

1989
(A1) How many base ten integers of the form 1010101 · · · 101 are prime?

(A3) Prove that all roots of 11z 10 + 10iz 9 + 10iz − 11 = 0 have unit modu-
lus (or equivalent |z| = 1).

(A4) Is there a gambling game with an honest coin for two players, in which the
probability of one of them winning is π1e .

(A5) Show that we can find α > 0 such that, given any point P inside a regular
2n + 1-gon which is inscribed in a circle radius 1, we can find two vertices of the
polygon whose distance from P differ by less than n1 − nα3 .

(A6) Let α = 1 + a1 x + a2 x2 + . . . be a formal power series with coefficients in


the field of two elements. Let
an = 1 if every block of zeroes in the binary expansion of has an even number
of zeroes
an = 0 otherwise
Prove that α3 + xα + 1 = 0.

(B1) A dart, thrown at random, hits a square target. Assuming that any two
parts of the target of equal area are equal likely to be hit, find the probability
that the point hit is nearer to the center than any edge.

Page 41
Putnam In Olympiad Mahdiyar Amid Sardari

(B2) Let S be a non-empty set with an associative operation that is left and
right consultative (xy = xz implies y = z, and yx = zx implies y = z). Assume
that for every a in S the set {an : n = 0, 1, 2, · · · } is finite. Must S be a group?

(B4) Can a countably infinite set have an uncountable collection of non-empty


subsets such that the intersection of any two of them is finite?

(B5) Label the vertices of a trapezoid T inscribed in the unit circle as A, B, C, D


counterclockwise with AB ∥ CD. Let s1 , s2 , and d denote the lengths of AB,
CD, and OE, where E is the intersection of the diagonals of T , and O is the
center of the circle. Determine the least upper bound of s1 −s d
2
over all T for
which d ̸= 0, and describe all cases, if any, in which equality is attained.

1988
(A1) Let R be the region consisting of the points (x, y) of the Cartesian plane
satisfying both |x| − |y| ≤ 1 and |y| ≤ 1. Sketch the region R and find its area.

(A4) 1) If every point of the plane is painted one of three colors, do there
necessarily exist two points of the same color exactly one inch apart?
2) What if ”three” is replaced by ”nine”?

(A5) Prove that there exists a unique function f from the set R+ of positive
real numbers to R+ such that

f (f (x)) = 6x − f (x)

and
f (x) > 0
for all x > 0.

(A6) If a linear transformation A on an n-dimensional vector space has n + 1


eigenvectors such that any n of them are linearly independent, does it follow
that A is a scalar multiple of the identity?

(B1) A composite (positive integer) is a product ab with a and b not neces-


sarily distinct integers in {2, 3, 4, . . . }. Show that every composite is expressible
as xy + xz + yz + 1, with x, y, z positive integers.

(B2) Prove or disprove: If x and y are real numbers with y ≥ 0 and y(y + 1) ≤
(x + 1)2 , then y(y − 1) ≤ x2 .

(B3) For every n in the set N = {1, 2, . . . } of positive integers, let rn be the

Page 42
Putnam In Olympiad Mahdiyar Amid Sardari


minimum value of |c − d 3| for all non negative integers c and d with c + d = n.
Find, with proof, the smallest positive real number g with rn ≤ g for all n ∈ N .
P∞
(B4) ProvePthat if n=1 an is a convergent series of positive real numbers,

then so is n=1 (an )n/(n+1) .

(B6) Prove that there exist an infinite number of ordered pairs (a, b) of in-
tegers such that for every positive integer t, the number at + b is a triangular
number if and only if t is a triangular number. (The triangular numbers are the
tn = n(n + 1)/2 with n in {0, 1, 2, . . . }.)

1987
(A1) Curves A, B, C and D are defined in the plane as follows:

 
x
A= (x, y) : x2 − y 2 = ,
x2 + y 2
 
y
B = (x, y) : 2xy + 2 = 3 ,
x + y2
C = (x, y) : x3 − 3xy 2 + 3y = 1 ,


D = (x, y) : 3x2 y − 3x − y 3 = 0 .


Prove that A ∩ B = C ∩ D.

(A2) The sequence of digits

123456789101112131415161718192021 . . .

is obtained by writing the positive integers in order. If the 10n -th digit in this
sequence occurs in the part of the sequence in which the m-digit numbers are
placed, define f (n) to be m. For example, f (2) = 2 because the 100th digit
enters the sequence in the placement of the two-digit integer 55. Find, with
proof, f (1987).

(A4) Let P be a polynomial, with real coefficients, in three variables and F


be a function of two variables such that

P (ux, uy, uz) = u2 F (y − x, z − x) for all real x, y, z, u,

and such that P (1, 0, 0) = 4, P (0, 1, 0) = 5, and P (0, 0, 1) = 6. Also let A, B, C


be complex numbers with P (A, B, C) = 0 and |B − A| = 10. Find |C − A|.

Page 43
Putnam In Olympiad Mahdiyar Amid Sardari

(A5) Let  
⃗ −y x
G(x, y) = , ,0 .
x2 + 4y 2 x2 + 4y 2
Prove or disprove that there is a vector-valued function

F⃗ (x, y, z) = (M (x, y, z), N (x, y, z), P (x, y, z))

with the following properties:


(1) M, N, P have continuous partial derivatives for all (x, y, z) ̸= (0, 0, 0);
(2) Curl F⃗ = ⃗0 for all (x, y, z) ̸= (0, 0, 0);
(3) F⃗ (x, y, 0) = G(x,
⃗ y).

(A6) For each positive integer n, let a(n) be the number of zeroes in the base 3
representation of n. For which positive real numbers x does the series

X xa(n)
n=1
n3

converge?

(B2) Let r, s and t be integers with 0 ≤ r, 0 ≤ s and r + s ≤ t. Prove that


s s s
  
0 1  s  t+1
t + t + ··· + t = .
r r+1 r+s (t + 1 − s) t−s
r

(B3) Let F be a field in which 1 + 1 ̸= 0. Show that the set of solutions to


the equation x2 + y 2 = 1 with x and y in F is given by (x, y) = (1, 0) and
 2 
r −1 2r
(x, y) = ,
r2 + 1 r2 + 1

where r runs through the elements of F such that r2 ̸= −1.

(B4) Let (x1 , y1 ) = (0.8, 0.6) and let xn+1 = xn cos yn − yn sin yn and yn+1 =
xn sin yn + yn cos yn for n = 1, 2, 3, . . . . For each of limn→∞ xn and limn→∞ yn ,
prove that the limit exists and find it or prove that the limit does not exist.

1986
(A1)Find, with explanation, the maximum value of f (x) = x3 − 3x on the set
of all real numbers x satisfying x4 + 36 ≤ 13x2 .

Page 44
Putnam In Olympiad Mahdiyar Amid Sardari

(A2) What is the units (i.e., rightmost) digit of

1020000
 
?
10100 + 3

P∞
(A3) Evaluate n=0 Arccot(n2 + n + 1), where Arccot t for t ≥ 0 denotes the
number θ in the interval 0 < θ ≤ π/2 with cot θ = t.

(A6) Let a1 , a2 , . . . , an be real numbers, and let b1 , b2 , . . . , bn be distinct positive


integers. Suppose that there is a polynomial f (x) satisfying the identity
n
X
(1 − x)n f (x) = 1 + ai xbi .
i=1

Find a simple expression (not involving any sums) for f (1) in terms of b1 , b2 , . . . , bn
and n (but independent of a1 , a2 , . . . , an ).

(B1) Inscribe a rectangle of base b and height h in a circle of radius one, and
inscribe an isosceles triangle in the region of the circle cut off by one base of the
rectangle (with that side as the base of the triangle). For what value of h do
the rectangle and triangle have the same area?

(B2) Prove that there are only a finite number of possibilities for the ordered
triple T = (x − y, y − z, z − x), where x, y, z are complex numbers satisfying the
simultaneous equations

x(x − 1) + 2yz = y(y − 1) + 2zx = z(z − 1) + 2xy,

and list all such triples T .

(B3) Let Γ consist of all polynomials in x with integer coefficients. For f and
g in Γ and m a positive integer, let f ≡ g (mod m) mean that every coefficient
of f − g is an integral multiple of m. Let n and p be positive integers with p
prime. Given that f, g, h, r and s are in Γ with rf + sg ≡ 1 (mod p) and f g ≡ h
(mod p), prove that there exist F and G in Γ with F ≡ f (mod p), G ≡ g
(mod p), and F G ≡ h (mod pn ).

(B4)
√ For a positive real number r, let G(r) be the minimum value of |r −
m2 + 2n2 | for all integers m and n. Prove or disprove the assertion that
limr→∞ G(r) exists and equals 0.

(B5) Let f (x, y, z) = x2 + y 2 + z 2 + xyz. Let p(x, y, z), q(x, y, z), r(x, y, z)
be polynomials with real coefficients satisfying

f (p(x, y, z), q(x, y, z), r(x, y, z)) = f (x, y, z).

Page 45
Putnam In Olympiad Mahdiyar Amid Sardari

Prove or disprove the assertion that the sequence p, q, r consists of some permu-
tation of ±x, ±y, ±z, where the number of minus signs is 0 or 2.

1985
(A1) Determine, with proof, the number of ordered triples (A1 , A2 , A3 ) of sets
which have the property that
(1) A1 ∪ A2 ∪ A3 = {1, 2, 3, 4, 5, 6, 7, 8, 9, 10}, and
(2) A1 ∩ A2 ∩ A3 = ∅.
Express your answer in the form 2a 3b 5c 7d , where a, b, c, d are non negative in-
tegers.

(A2) Let T be an acute triangle. Inscribe a rectangle R in T with one side


along a side of T. Then inscribe a rectangle S in the triangle formed by the side
of R opposite the side on the boundary of T, and the other two sides of T, with
one side along the side of R. For any polygon X, let A(X) denote the area of
X. Find the maximum value, or show that no maximum exists, of A(R)+A(S) A(T ) ,
where T ranges over all triangles and R, S over all rectangles as above.

(A3) Let d be a real number. For each integer m ≥ 0, define a sequence


{am (j)} , j = 0, 1, 2, . . . by the condition
am (0) = d/2m ,
2
am (j + 1) = (am (j)) + 2am (j), j ≥ 0.
Evaluate limn→∞ an (n).

(A4) Define a sequence {ai } by a1 = 3 and ai+1 = 3ai for i ≥ 1. Which in-
tegers between 00 and 99 inclusive occur as the last two digits in the decimal
expansion of infinitely many ai ?

(A6) If p(x) = a0 + a1 x + · · · + am xm is a polynomial with real coefficients


ai , then set
Γ(p(x)) = a20 + a21 + · · · + a2m .
Let F (x) = 3x2 + 7x + 2. Find, with proof, a polynomial g(x) with real coeffi-
cients such that
(1) g(0) = 1, and
(2) Γ (f (x)n ) = Γ (g(x)n )
for every integer n ≥ 1.

(B1) Let k be the smallest positive integer for which there exist distinct in-
tegers m1 , m2 , m3 , m4 , m5 such that the polynomial
p(x) = (x − m1 ) (x − m2 ) (x − m3 ) (x − m4 ) (x − m5 )

Page 46
Putnam In Olympiad Mahdiyar Amid Sardari

has exactly k nonzero coefficients. Find, with proof, a set of integers m1 , m2 , m3 , m4 , m5


for which this minimum k is achieved.

(B3) Let
a1,1 a1,2 a1,3 ...
a2,1 a2,2 a2,3 ···
a3,1 a3,2 a3,3 ···
.. .. .. ..
. . . .
be a doubly infinite array of positive integers, and suppose each positive integer
appears exactly eight times in the array. Prove that am,n > mn for some pair
of positive integers (m, n).

(B4) Let C be the unit circle x2 + y 2 = 1. A point p is chosen randomly on


the circumference C and another point q is chosen randomly from the interior
of C (these points are chosen independently and uniformly over their domains).
Let R be the rectangle with sides parallel to the x and y-axes with diagonal pq.
What is the probability that no point of R lies outside of C?

1984
(A1) Let A be a solid a × b × c rectangular brick, where a, b, c > 0. Let B be
the set of all points which are a distance of at most one from some point of A.
Express the volume of B as a polynomial in a, b, and c.

6k
P∞
(A2) Express k=1 (3k+1 −2k+1 )(3k −2k ) as a rational number.

(A4) A convex pentagon P = ABCDE is inscribed in a circle of radius 1.


Find the maximum area of P subject to the condition that the chords AC and
BD are perpendicular.

(A6) Let n be a positive integer, and let f (n) denote the last nonzero digit
in the decimal expansion of n!.
(1) Show that if a1 , a2 , . . . , ak are distinct non negative integers, then f (5a1 +
5a2 + . . . + 5ak ) depends only on the sum a1 + a2 + . . . + ak .
(2) Assuming part (a), we can define
g(s) = f (5a1 + 5a2 + . . . + 5ak ),
where s = a1 + a2 + . . . + ak . Find the least positive integer p for which
g(s) = g(s + p), for all s ≥ 1,
or show that no such p exists.

Page 47
Putnam In Olympiad Mahdiyar Amid Sardari

(B1) Let n be a positive integer, and define f (n) = 1! + 2! + . . . + n!. Find


polynomials P and Q such that
f (n + 2) = P (n)f (n + 1) + Q(n)f (n)
for all n ≥ 1.

(B2) Find the minimum value of


p 2
9
(u − v)2 + 2 − u2 −
v

for 0 < u < 2 and v > 0.

(B3) Prove or disprove the following statement: If F is a finite set with two
or more elements, then there exists a binary operation ∗ on F such that for all
x, y, z in F ,
(1) x ∗ z = y ∗ z implies x = y
(2) x ∗ (y ∗ z) ̸= (x ∗ y) ∗ z

(B4) Find, with proof, all real-valued functions y = g(x) defined and continuous
on [0, ∞), positive on (0, ∞), such that for all x > 0 the y-coordinate of the
centroid of the region
Rx = {(s, t) | 0 ≤ s ≤ x, 0 ≤ t ≤ g(s)}
is the same as the average value of g on [0, x].

(B5) For each non negative integer k, let d(k) denote the number of 1′ s in
the binary expansion of k. Let m be a positive integer. Express
m
2X −1
(−1)d(k) k m
k=0

in the form (−1)m af (m) g(m)!, where a is an integer and f and g are polynomials.

(B6) A sequence of convex polygons (Pn ), n ≥ 0, is defined inductively as follows.


P0 is an equilateral triangle with side length 1. Once Pn has been determined,
its sides are trisected; the vertices of Pn+1 are the√ interior trisection points of
the sides of Pn . Express limn→∞ [Pn ] in the form ba , where a, b are integers.

1983
(A1) How many positive integers n are there such that n is an exact divisors of
at least one of the numbers 1040 and 2030 ?

Page 48
Putnam In Olympiad Mahdiyar Amid Sardari

(A2) The shorthand of a clock has the length 3, the longhand has the length 4.
Determine the distance between the endpoints of the hands at the time, where
their distance increases the most.

(A3) Let p be an odd prime and let


F (n) = 1 + 2n + 3n2 + . . . + (p − 1)np−2 .
Prove that if a and b are distinct integers in {0, 1, 2, . . . , p − 1} then F (a) and
F (b) are not congruent modulo p.

(A4) Let k be a positive integer and let m = 6k − 1. Let


2k−1  
X m
S(m) = (−1)j+1 .
j=1
3j − 1

Prove that S(m) is never zero.

(A5) Prove or disprove that there exists a positive real u such that ⌊un ⌋ − n is
an even integer for all positive integers n.

(B1) Let v be a vertex of a cube C with edges of length 4. Let S be the


largest sphere that can be inscribed in C. Let R be the region consisting of all
points p between S and C such that p is closer to v than to any other vertex of
the cube. Find the volume of R.

(B2) For positive integers n, let C(n) be the number of representation of n


as a sum of non increasing powers of 2, where no power can be used more than
three times. For example, C(8) = 5 since the representations of 8 are:
8, 4 + 4, 4 + 2 + 2, 4 + 2 + 1 + 1, and 2 + 2 + 2 + 1 + 1.
Prove or disprove that there is a polynomial P (x) such that C(n) = ⌊P (n)⌋ for
all positive integers n.

(B4) Problem. Let f : R0+ → R0+ be a function defined as



f (n) = n + ⌊ n⌋ ∀ n ∈ R0+ .
Prove that for any positive integer m, the sequence
m, f (m), f (f (m)), f (f (f (m))), . . .
contains a perfect square.

(B6) Let k be a positive integer, let m = 2k + 1, and let r ̸= 1 be a com-


plex root of z m − 1 = 0. Prove that there exist polynomials P (z) and Q(z) with
integer coefficients such that (P (r))2 + (Q(r))2 = −1.

Page 49
Putnam In Olympiad Mahdiyar Amid Sardari

1982
(A1) Let V be the region in the Cartesian plane consisting of all points (x, y)
satisfying the simultaneous conditions

|x| ≤ y ≤ |x| + 3 and y ≤ 4.

Find the centroid of V .

(A5) a, b, c, d are positive integers, and r = 1 − ab − dc . And, a + c ≤ 1982, r ≥ 0.


1
Prove that r > 1983 3.

(A6) Let σ be a bijection on the positive integers. Let x1 , x2 , x3 , . . . be a se-


quence of real numbers with the following three properties:
(1) |xn | is a strictly decreasing function of n;
(2) |σ(n) − n| P·n|xn | → 0 as n → ∞;
(3) limn→∞ k=1 xk = 1.
Prove or disprove that these conditions imply that
n
X
lim xσ(k) = 1.
n→∞
k=1

(B1) Let M be the midpoint of side BC of △ABC. Using the smallest pos-
sible n, described a method for cutting △AM B into n triangles which can be
reassembled to form a triangle congruent to △AM C.

(B3) Let pn be the probability that c + d is a perfect square when the inte-
√ independently at random from the set {1, 2,√. . . , n}.
gers c and d are selected
Show that limn→∞ pn n exists and express this limit in the form r( s − t),
where s and t are integers and r is a rational number.

(B4) Let n1 , n2 , . . . , ns be distinct integers such that

(n1 + k)(n2 + k) · · · (ns + k)

is an integral multiple of n1 n2 · · · ns for every integer k. For each of the following


assertions give a proof or a counterexample:
(1) |ni | = 1 for some i
(2) If further all ni are positive, then

{n1 , n2 , . . . , n2 } = {1, 2, . . . , s}.

(B6) Denote by S(a, b, c) the area of a triangle whose lengths of three sides

Page 50
Putnam In Olympiad Mahdiyar Amid Sardari

are a, b, c Prove that for any positive real numbers a1 , b1 , c1 and a2 , b2 , c2 which
can serve as the lengths of three sides of two triangles respectively ,we have
p p p
S(a1 , b1 , c1 ) + S(a2 , b2 , c2 ) ≤ S(a1 + a2 , b1 + b2 , c1 + c2 ).

1981
(A1) Let E(n) denote the largest integer k such that 5k divides 11 ·22 ·33 ·. . .·nn .
Calculate
E(n)
lim .
n→∞ n2

(A2) Two distinct squares of the 8 × 8 chessboard C are said to be adjacent if


they have a vertex or side in common. Also, g is called a C-gap if for every
numbering of the squares of C with all the integers 1, 2, . . . , 64 there exist two
adjacent squares whose numbers differ by at least g. Determine the largest C-
gap g.

(A4) A point P moves inside a unit square in a straight line at unit speed.
When it meets a corner it escapes. When it meets an edge its line of motion is
reflected so that the angle of incidence equals the angle of reflection. Let N (t)
be the number of starting directions from a fixed interior point P0 for which P
escapes within t units of time. Find the least constant a for which constants b
and c exist such that
N (t) ≤ at2 + bt + c
for all t > 0 and all initial points P0 .

(A5) Let P (x) be a polynomial with real coefficients and form the polynomial

Q(x) = (x2 + 1)P (x)P ′ (x) + x(P (x)2 + P ′ (x)2 ).

Given that the equation P (x) = 0 has n distinct real roots exceeding 1, prove
or disprove that the equation Q(x) = 0 has at least 2n − 1 distinct real roots.

(A6) Suppose that each of the vertices of ABC is a lattice point in the xy-
plane and that there is exactly one lattice point P in the interior of the triangle.
The line AP is extended to meet BC at E. Determine the largest possible value
for the ratio of lengths of segments

|AP |
.
|P E|

Page 51
Putnam In Olympiad Mahdiyar Amid Sardari

(B1) Find
n n
1 XX 4
lim (5h − 18h2 k 2 + 5k 4 ).
n→∞ n5
h=1 k=1

(B2) Determine the minimum value of


s 2  2  2
t 4
(r − 1)2 + −1 + −1 + −1
r s t

for all real numbers 1 ≤ r ≤ s ≤ t ≤ 4.

(B3) Prove that there are √ infinitely many positive n that for all prime divi-
sors p of n2 + 3, ∃ 0 ≤ k ≤ n and p | k 2 + 3.

(B5) Let B(n) be the number of ones in the base two expression for the positive
integer n. Determine whether

!
X B(n)
exp
n=1
n(n + 1)

is a rational number.

1980
(A1) Let b and c be fixed real numbers and let the ten points (j, yj ) for j =
1, 2, . . . , 10 lie on the parabola y = x2 + bx + c. For j = 1, 2, . . . , 9 let Ij be the
intersection of the tangents to the given parabola at (j, yj ) and (j + 1, yj+1 ).
Determine the polynomial function y = g(x) of least degree whose graph passes
through all nine points Ij .

(A2) Let r and s be positive integers. Derive a formula for the number of
ordered quadruples (a, b, c, d) of positive integers such that

3r · 7s = lcm(a, b, c) = lcm(a, b, d) = lcm(a, c, d) = lcm(b, c, d),

depending only on r and s.

(A4) 1) Prove that there exist integers a, b, c not all zero and each of absolute
value less than one million, such that
√ √
|a + b 2 + c 3| < 10−11 .

Page 52
Putnam In Olympiad Mahdiyar Amid Sardari

2) Let a, b, c be integers, not all zero and each of absolute value less than one
million. Prove that √ √
|a + b 2 + c 3| > 10−21 .

(B1) For which real numbers c is


ex + e−x 2
≤ ecx
2
for all real x?

(B2) Let S be the solid in three-dimensional space consisting of all points (x, y, z)
satisfying the following six simultaneous conditions:
x, y, z ≥ 0, x + y + z ≤ 11, 2x + 4y + 3z ≤ 36, 2x + 3z ≤ 44.

1) Determine the number V of vertices of S.


2) Determine the number E of edges of S.
3) Sketch in the bc-plane the set of points (b, c) such that (2, 5, 4) is one of the
points (x, y, z) at which the linear function bx + cy + z assumes its maximum
value on S.

(B3) For which real numbers a does the sequence (un ) defined by the initial
condition u0 = a and the recursion un+1 = 2un − n2 have un > 0 for all n ≥ 0?

(B4) Let A1 , A2 , . . . , A1066 be subsets of a finite set X such that |Ai | > 21 |X|
for 1 ≤ i ≤ 1066. Prove that there exist ten elements x1 , x2 , . . . , x10 of X such
that every Ai contains at least one of x1 , x2 , . . . , x10 .

(B5) For each t ≥ 0 let St be the set of all non negative, increasing, con-
vex, continuous, real-valued functions f (x) defined on the closed interval [0, 1]
for which
f (1) − 2f (2/3) + f (1/3) ≥ t(f (2/3) − 2f (1/3) + f (0)).
Define necessary and sufficient conditions on t for St to be closed under multi-
plication.

(B6) An infinite array of rational numbers G(d, n) is defined for integers d and
n with 1 ≤ d ≤ n as follows:
n
1 dX
G(1, n) = , G(d, n) = G(d − 1, i − 1) for d > 1.
n n
i=d

For 1 < d < p and p prime, prove that G(d, p) is expressible as a quotient s/t
of integers s and t with t not divisible by p.

Page 53
Putnam In Olympiad Mahdiyar Amid Sardari

1979
(A1) Find positive integers n and a1 , a2 , . . . , an such that

a1 + a2 + . . . an = 1979

and the product a1 a2 . . . an as large as possible.

(A2) Establish necessary and sufficient conditions on the constant k for the
existence of a continuous real valued function f (x) satisfying

f (f (x)) = kx9

for all real x.

(A3) Let x1 , x2 , x3 , . . . be a sequence of nonzero real numbers satisfying


xn−2 xn−1
xn = for n = 3, 4, 5, . . . .
2xn−2 − xn−1

Establish necessary and sufficient conditions on x1 and x2 for xn to be an inte-


ger for infinitely many values of n.

(A4) Let A be a set of 2n points in the plane, no three of which are collinear.
Suppose that n of them are colored red and the remaining n blue. Prove or
disprove: there are n closed straight line segments, no two with a point in com-
mon, such that the endpoints of each segment are points of A having different
colors.

(A5) Denote by ⌈x⌉ the greatest integer less than or equal to x and by S(x) the
sequence ⌈x⌉, ⌈2x⌉, ⌈3x⌉, . . . . Prove that there are distinct real solutions α and
β of the equation
x3 − 10x2 + 29x − 25 = 0
such that infinitely many positive integers appear both in S(α) and in S(β).

(A6) Let 0 ≤ pi ≤ 1 for i = 1, 2, . . . , n. Show that


n
X 1 1
≤ 8n(1 + 1/3 + 1/5 + · · · + )
i=1
|x − pi | 2n − 1

for some x satisfying 0 ≤ x ≤ 1.

(B1) Prove or disprove: there is at least one straight line normal to the graph
of y = cosh x at a point (a, cosh a) and also normal to the graph of y = sinh x
at a point (c, sinh c).

(B5) In the plane, let C be a closed convex set that contains (0, 0) but no

Page 54
Putnam In Olympiad Mahdiyar Amid Sardari

other point with integer coordinates. Suppose that A(C), the area of C, is
equally distributed among the four quadrants. Prove that A(C) ≤ 4.

(B6)√For k = 1, 2 . . . , n let zk = xk + iyk , where the xk and yk are real and


i = −1. Let r bet the absolute value of the real part of
q
± z12 + z22 + . . . zn2 .

Prove that r ≤ |x1 | + |x2 | + · · · + |xn |.

1978
(A1) Let A be any set of 20 distinct integers chosen from the arithmetic progres-
sion 1, 4, 7, . . . , 100. Prove that there must be two distinct integers in A whose
sum is 104.

(A4) A bypass operation on a set S is a mapping B : S × S → S with the


property B(B(w, x), B(y, z)) = B(w, z) for all w, x, y, z ∈ S.
(1) Prove that B(a, b) = c implies B(c, c) = c when B is a bypass.
(2) Prove that B(a, b) = c implies B(a, x) = B(c, x) for all x ∈ S when B is a
bypass.
(3) Construct a bypass operation B on a finite set S with the following three
properties
1) B(x, x) = x for all x ∈ S.
2) There exist d and e in S with B(d, e) = d ̸= e.
3) There exist f and g in S with B(f, g) ̸= f.

(A5) Let 0 < xi < π for i = 1, 2, . . . , n and set


x1 + x2 + . . . + xn
x= .
n
Prove that
n  n
Y sin xi sin x
≤ .
i=1
xi x

(A6) Let n distinct points in the plane be given. Prove that fewer than 2n3/2
pairs of them are a unit distance apart.

(B1) Find the area of a convex octagon that is inscribed in a circle and has
four consecutive sides of length
√ 3 and the remaining four sides of length 2. Give
the answer in the form r + s t with r, s, t positive integers.

Page 55
Putnam In Olympiad Mahdiyar Amid Sardari

(B2) Express
∞ X

X 1
n=1 m=1
m2 n + mn2 + 2mn
as a rational number.

(B3) The sequence (Qn (x)) of polynomials is defined by

Q1 (x) = 1 + x, Q2 (x) = 1 + 2x,

and for m ≥ 1 by

Q2m+1 (x) = Q2m (x) + (m + 1)xQ2m−1 (x),

Q2m+2 (x) = Q2m+1 (x) + (m + 1)xQ2m (x).


Let xn be the largest real root of Qn (x). Prove that (xn ) is an increasing se-
quence and that limn→∞ xn = 0.

(B4) Prove that for every real number N the equation

x21 + x22 + x23 + x24 = x1 x2 x3 + x1 x2 x4 + x1 x3 x4 + x2 x3 x4

has an integer solution (x1 , x2 , x3 , x4 ) for which x1 , x2 , x3 and x4 are all larger
than N.

(B5) Find the largest a for which there exists a polynomial

P (x) = ax4 + bx3 + cx2 + dx + e

with real coefficients which satisfies 0 ≤ P (x) ≤ 1 for −1 ≤ x ≤ 1.

(B6) Let p and n be positive integers. Suppose that the numbers chk (h =
1, 2, . . . , n ; k = 1, 2, . . . , ph) satisfy 0 ≤ chk ≤ 1. Prove that
X c 2
hk
X
≤ 2p chk ,
h
where each summation is over all admissible ordered pairs (h, k).

1977
(A1) Consider all lines which meet the graph of

y = 2x4 + 7x3 + 3x − 5

Page 56
Putnam In Olympiad Mahdiyar Amid Sardari

in four distinct points, say (xi , yi ), i = 1, 2, 3, 4. Show that


x1 + x2 + x3 + x4
4
is independent of the line and find its value.

(A2) Determine all solutions in real numbers x, y, z, w of the system


1 1 1 1
x + y + z = w, + + = .
x y z w

(A3) Let u, f and g be functions, defined for all real numbers x, such that

u(x + 1) + u(x − 1) u(x + 4) + u(x − 4)


= f (x) and = g(x).
2 2
Determine u(x) in terms of f and g.

(A4) For 0 < x < 1, express


∞ n
X x2
n=0
1 − x2n+1
as a rational function of x.

(A5) Prove that    


pa a
= (mod p)
pb b
for all integers p, a, and b with p a prime, p > 0, and a > b > 0.

(B1) Evaluate the infinite product



Y n3 − 1
.
n=2
n3 + 1

(B2) Given a convex quadrilateral ABCD and a point O not in the plane
ABCD, locate point A′ on line OA, point B ′ on the line OB, point C ′ on
line OC, and point D′ on line OD so that A′ B ′ C ′ D′ is a parallelogram.

(B3) An (ordered) triple (x1 , x2 , x3 ) of positive irrational numbers with x1 +


x2 + x3 = 1 is called balanced if each xi < 1/2. If a triple is not balanced, say
if xj > 1/2, one performs the following balancing act

B(x1 , x2 , x3 ) = (x′1 , x′2 , x′3 ),

Page 57
Putnam In Olympiad Mahdiyar Amid Sardari

where x′i = 2xi if i ̸= j and x′j = 2xj − 1. If the new triple is not balanced, one
performs the balancing act on it. Does the continuation of this process always
lead to a balanced triple after a finite number of performances of the balancing
act?

(B4) Let C be a continuous closed curve in the plane which does not cross
itself and let Q be a point inside C. Show that there exists points P1 and P2
on C such that Q is the midpoint of the line segment P1 P2 .

(B5) Suppose that a1 , a2 , . . . an are real (n > 1) and


n n
X 1 X 2
A+ a2i < ( ai ) .
i=1
n − 1 i=1

Prove that A < 2ai aj for 1 ≤ i < j ≤ n.

(B6) Let H be a subgroup with h elements in a group G. Suppose that G


has an element a such that for all x in H, (xa)3 = 1, the identity. In G, let P
be the subset of all products x1 ax2 a . . . xn a, with n a positive integer and the
xi in H.
(1) Show that P is a finite set.
(2) Show that, in fact, P has no more that 3h2 elements.

1976
(A1) P is an interior point of the angle whose sides are the rays OA and OB.
Locate X on OA and Y on OB so that the line segment XY contains P and so
that the product (P X)(P Y ) is a minimum.

(A2) Let P (x, y) = x2 y + xy 2 and Q(x, y) = x2 + xy + y 2 . For n = 1, 2, 3, . . . ,


let

Fn (x, y) = (x + y)n − xn − y n and,


Gn (x, y) = (x + y)n + xn + y n .

One observes that

G2 = 2Q, F3 = 3P, G4 = 2Q2 , F5 = 5P Q, G6 = 2Q3 + 3P 2 .

Prove that, in fact, for each n either Fn or Gn is expressible as a polynomial in


P and Q with integer coefficients.

(A3) Find all integral solutions of the equation

|pr − q s | = 1,

Page 58
Putnam In Olympiad Mahdiyar Amid Sardari

where p and q are prime numbers and r and s are positive integers larger than
unity. Prove that there are no other solutions.

(A4) Let r be a root of P (x) = x3 + ax2 + bx − 1 = 0 and r + 1 be a root


of y 3 + cy 2 + dy + 1 = 0, where a, b, c and d are integers. Also let P (x) be
irreducible over the rational numbers. Express another root s of P (x) = 0 as a
function of r which does not explicitly involve a, b, c or d.

(B2) Suppose that G is a group generated by elements A and B, that is, ev-
ery element of G can be written as a finite ”word” An1 B n2 An3 . . . B nk , where
n1 , . . . nk are any integers, and A0 = B 0 = 1 as usual. Also suppose that
A4 = B 7 = ABA−1 B = 1, A2 ̸= 1, and B ̸= 1.
(1) How many elements of G are of the form C 2 with C in G?
(2) Write each such square as a word in A and B.

(B3) Suppose that we have n events A1 , . . . , An , each of which has probabil-


ity at least 1 − a of occurring, where a < 1/4. Further suppose that Ai and
Aj are mutually independent if |i − j| > 1. Assume as known that the recur-
rence uk+1 = uk − auk−1 , u0 = 1, u1 = 1 − a, defines positive real numb uk for
k = 0, 1, . . . . Show that the probability of all of A1 , . . . , An occurring is at least
un .

(B4) For a point P on an ellipse, let d be the distance from the center of
the ellipse to the line tangent to the ellipse at P. Prove that (P F1 )(P F2 )d2 is
constant as P varies on the ellipse, where P F1 and P F2 are distances from P
to the foci F1 and F2 of the ellipse.

(B5) Evaluate
n  
X
kn
(−1) (x − k)n .
k
k=0

(B6) As usual, let σ(N ) denote the sum of all the (positive integral) divisors
of N. (Included among these divisors are 1 and N itself.) For example, if p is
a prime, then σ(p) = p + 1. Motivated by the notion of a ”perfect” number, a
positive integer N is called ”quasiperfect” if σ(N ) = 2N + 1. Prove that every
quasiperfect number is the square of an odd integer.

1975
(A1) Show that a positive integer m is a sum of two triangular numbers if and
only if 4m + 1 is a sum of two squares.

Page 59
Putnam In Olympiad Mahdiyar Amid Sardari

(A2) Describe the region R consisting of the points (a, b) of the cartesian plane
for which both (possibly complex) roots of the polynomial z 2 + az + b have
absolute value smaller than 1.

(A3) Let 0 < α < β < γ ∈ R. Let


K = {(x, y, z) ∈ R3 | x, y, z ≥ 0 and xβ + y β + z β = 1}.
Define f : K → R, (x, y, z) 7→ xα + y β + z γ . At which points of K does f
assume its minimal and maximal values?

(A4) Let m > 1 be an odd integer. Let n = 2m and θ = e2πi/n . Find in-
Pk 1
tegers a1 , . . . , ak such that i=1 ai θi = 1−θ .

(A6) Given three points in space forming an acute-angled triangle, show that
we can find two further points such that no three of the five points are collinear
and the line through any two is normal to the plane through the other three.

(B1) Consider the additive group Z 2 . Let H be the smallest subgroup con-
taining (3, 8), (4, −1) and (5, 4). Let Hxy be the smallest subgroup containing
(0, x) and (1, y). Find some pair (x, y) with x > 0 such that H = Hxy .

(B2) A slab is the set of points strictly between two parallel planes. Prove
that a countable sequence of slabs, the sum of whose thicknesses converges,
cannot fill space.

(B4) Does a circle have a subset which is topologically closed and which contains
exactly one point of each pair of diametrically opposite points?
Pn 1
(B6) Let Hn = r=1 r . Show that

n − (n − 1)n−1/(n−1) > Hn > n(n + 1)1/n − n

for n > 2.

1974
(A1) Call a set of positive integers ”conspiratorial” if no three of them are
pairwise relatively prime. What is the largest number of elements in any ”con-
spiratorial” subset of the integers 1 to 16?

(A2) A circle stands in a plane perpendicular to the ground and a point A


lies in this plane exterior to the circle and higher than its bottom. A particle
starting from rest at A slides without friction down an inclined straight line
until it reaches the circle. Which straight line allows descent in the shortest

Page 60
Putnam In Olympiad Mahdiyar Amid Sardari

time?

(A3) A well-known theorem asserts that a prime p > 2 can be written as the
sum of two perfect squares (p = m2 + n2 , with m and n integers) if and only
if p ≡ 1 (mod 4). Assuming this result, find which primes p > 2 can be written
in each of the following forms, using integers x and y:
1) x2 + 16y 2 ,
2) 4x2 + 4xy + 5y 2 .

(A4) An unbiased coin is tossed n times. What is the expected value of |H − T |,


where H is the number of heads and T is the number of tails?

(A5) Consider the two mutually tangent parabolas y = x2 and y = −x2 . The
upper parabola rolls without slipping around the fixed lower parabola. Find the
locus of the focus of the moving parabola.

(A6) Given n, let k = k(n) be the minimal degree of any monic integral poly-
nomial
f (x) = xk + ak−1 xk−1 + . . . + a0
such that the value of f (x) is exactly divisible by n for every integer x. Find
the relationship between n and k(n). In particular, find the value of k(n) cor-
responding to n = 106 .

(B1) Which configurations of five (not necessarily distinct) points p1 , . . . , p5


on the circle x2 + y 2 = 1 maximize the sum of the ten distances
X
d(pi , pj )?
i<j

(B3) Prove that if a is a real number such that


1
cos πa = ,
3
then a is irrational.

(B4) A function f : R2 → R is said to be continuous in each variable sepa-


rately if, for each fixed value y0 of y, the function f (x, y0 ) is continuous in the
usual sense as a function in x, and similarly f (x0 , y) is continuous as a function
of y for each fixed x0 . Let f : R2 → R be continuous in each variable separately.
Show that there exists a sequence of continuous functions gn : R2 → R such
that
f (x, y) = lim gn (x, y)
n→∞

for all

Page 61
Putnam In Olympiad Mahdiyar Amid Sardari

(B5) Show that


n n2 nn en
1+ + + ... + >
1! 2! n! 2
for every integer n ≥ 0.

(B6) For a set with n elements, how many subsets are there whose cardinally is
respectively ≡ 0 (mod 3), ≡ 1 (mod 3), ≡ 2 (mod 3)? In other words, calculate
 
X n
si,n =
k
k≡i (mod 3)

for i = 0, 1, 2. Your result should be strong enough to permit direct evaluation


of the numbers si,n and to show clearly the relationship of s0,n , s1,n and s2,n
to each other for all positive integers n. In particular, show the relationships
among these three sums for n = 1000.

1973
(A1) (1) Let ABC be any triangle. Let X, Y, Z be points on the sides BC, CA, AB
respectively. Suppose that BX ≤ XC, CY ≤ Y A, AZ ≤ ZB. Show that the
area of the triangle XY Z ≥ 1/4 times the area of ABC.
(2) Let ABC be any triangle, and let X, Y, Z be points on the sides BC, CA, AB
respectively. Using (a) or by any other method, show: One of the three corner
triangles AZY, BXZ, CY X has an area ≤ area of the triangle XY Z.

(A2) Consider an infinite series whose n-th term is ±(1/n), the ± signs be-
ing determined according to a pattern that repeats periodically in blocks of
eight (there are 28 possible patterns).
(1) Show that a sufficient condition for the series to be conditionally convergent
is that there are four ”+” signs and four ”−” signs in the block of eight signs.
(2) Is this sufficient condition also necessary?

(A3) Let n be a fixed positive integer and let b(n) be the minimum value of
n
k+ ,
k
where
√ k is allowed to range through all positive integers. Prove that ⌊b(n)⌋ =
⌊ 4n + 1⌋.

(A4) How many zeroes does the function f (x) = 2x −1−x2 have on the real line?

(A6) Prove that it is impossible for seven distinct straight lines to be situ-
ated in the euclidean plane so as to have at least six points where exactly three

Page 62
Putnam In Olympiad Mahdiyar Amid Sardari

of these lines intersect and at least four points where exactly two of these lines
intersect.

(B1) Let a1 , a2 , . . . a2n+1 be a set of integers such that, if any one of them
is removed, the remaining ones can be divided into two sets of n integers with
equal sums. Prove a1 = a2 = · · · = a2n+1 .

(B2) Let z = x + yi be a complex number with x and y rational and with


|z| = 1. Prove that the number |z 2n − 1| is rational for every integer n.

(B3) Consider an integer p > 1 with the property that the polynomial x2 − x + p
takes prime values for all integers x such that 0 ≤ x < p. Show that there is
exactly one triple of integers a, b, c satisfying the conditions:

b2 − 4ac = 1 − 4p, 0 < a ≤ c, −a ≤ b < a.

(B5) (1) Let z be a solution of the quadratic equation

az 2 + bz + c = 0

and let n be a positive integer. Show that z can be expressed as a rational


function of z n , a, b, c.
(2) Using (1) or by any other means, express x as a rational function of x3 and
x + x1 .

1972
n n n n
   
(A1) Show that m , m+1 , m+2 and m+3 cannot be in arithmetic progres-
sion, where n, m > 0 and n ≥ m + 3.

(A2) Let S be a set with a binary operation ∗ such that


1) a ∗ (a ∗ b) = b for all a, b ∈ S.
2) (a ∗ b) ∗ b = a for all a, b ∈ S. Show that ∗ is commutative and give an
example where ∗ is not associative.

(A4) Show that a circle inscribed in a square has a larger perimeter than any
other ellipse inscribed in the square.

(A5) Prove that there is no positive integer n > 1 such that n | 2n − 1.


P∞ n 2n P∞
(B1) Let n=0 x (x−1)
n! = n
n=0 an x . Show that no three consecutive an
can be equal to 0.

Page 63
Putnam In Olympiad Mahdiyar Amid Sardari

(B3) A group G has elements g, h satisfying ghg = hg 2 h, g 3 = 1 and hn = 1 for


some odd integer n. Prove that h = e, where e is the identity element.

(B4) Show that for n > 1 we can find a polynomial P (a, b, c) with integer
coefficients such that
P (xn , xn+1 , x + xn+2 ) = x.

(B5) Let A, B, C and D be non-coplanar points such that ̸ ABC = ̸ ADC


and ̸ BAD = ̸ BCD. Show that AB = CD and AD = BC.

(B6) Let n1 < n2 < n3 < · · · < nk be a set of positive integers. Prove that √
the
polynomial 1 + z n1 + z n2 + · · · + z nk has no roots inside the circle |z| < 5−1
2 .

1971
(A1) Let there be given nine lattice points (points with integral coordinates) in
three dimensional Euclidean space. Show that there is a lattice point on the
interior of one of the line segments joining two of these points.

(A2) Determine all polynomials P (x) such that P (x2 + 1) = (P (x))2 + 1 and
P (0) = 0.

(A3) The three vertices of a triangle of sides a, b, and c are lattice points and
lie on a circle of radius R. Show that abc ≥ 2R. (Lattice points are points in
Euclidean plane with integral coordinates.)

(A4) Show that for 0 < ϵ < 1 the expression (x + y)n (x2 − (2 − ϵ)xy + y 2 )
is a polynomial with positive coefficients for n sufficiently large and integral.
For ϵ = .002 find the smallest admissible value of n.

(A5) A game of solitaire is played as follows. After each play, according to


the outcome, the player receives either a or b points (a and b are positive in-
tegers with a greater than b), and his score accumulates from play to play. It
has been noticed that there are thirty-five non-attainable scores and that one
of these is 58. Find a and b.

(A6) Let c be a real number such that nc is an integer for every positive integer
n. Show that c is a non-negative integer.

Page 64
Putnam In Olympiad Mahdiyar Amid Sardari

(B1) Let S be a set and let ◦ be a binary operation on S satisfying two laws

x ◦ x = x for all x in S, and

(x ◦ y) ◦ z = (y ◦ z) ◦ x for all x, y, z in S.
Show that ◦ is associative and commutative.

(B2) Let F (x) be a real valued function defined for all real x except for x = 0
and x = 1 and satisfying the functional equation F (x) + F {(x − 1)/x} = 1 + x.
Find all functions F (x) satisfying these conditions.

(B3) Two cars travel around a track at equal and constant speeds, each com-
pleting a lap every hour. From a common starting point, the first starts at time
t = 0 and the second at an arbitrary later time t = T > 0. Prove that there
is a total period of exactly one hour during the motion in which the first has
completed twice as many laps as the second.

(B4) A ”spherical ellipse” with foci A, B on a given sphere is defined as the


⌢ ⌢ ⌢
set of all points P on the sphere such that P A + P B = constant. Here P A
denotes the shortest distance on the sphere between P and A. Determine the
entire class of real spherical ellipses which are circles.

(B6)
Px Let δ(x) be the greatest odd divisor of the positive integer x. Show that
| n=1 δ(n)/n − 2x/3| < 1, for all positive integers x.

1970
(A1) Show that the power series for the function

eax cos bx,

where a, b > 0, has either no zero coefficients or infinitely many zero coefficients.

(A2) Consider the locus given by the real polynomial equation

Ax2 + Bxy + Cy 2 + Dx3 + Ex2 y + F xy 2 + Gy 3 = 0,

where B 2 − 4AC < 0. Prove that there is a positive number δ such that there
are no points of the locus in the punctured disk

0 < x2 + y 2 < δ 2 .

Page 65
Putnam In Olympiad Mahdiyar Amid Sardari

(A3) Note that 122 = 144 ends in two 4s and 382 = 1444 end in three 4s.
Determine the length of the longest string of equal nonzero digits in which the
square of an integer can end.

(A4) Given a sequence (xn ) such that limn→∞ xn − xn−2 = 0, prove that
xn − xn−1
lim = 0.
n→∞ n

(A5) Determine the radius of the largest circle which can lie on the ellipsoid

x2 y2 z2
+ + =1 (a > b > c).
a2 b2 c2

(A6) Three numbers are chosen independently at random, one from each of the
three intervals [0, Li ] (i = 1, 2, 3). If the distribution of each random number is
uniform with respect to the length of the interval it is chosen from, determine
the expected value of the smallest number chosen.

(B1) Evaluate
2n
1 Y 2 1
lim (n + i2 ) n .
n→∞ n4
i=1

(B2) The time-varying temperature of a certain body is given by a polyno-


mial in the time of degree at most three. Show that the average temperature of
the body between 9 am and 3 pm can always be found by taking the average of
the temperatures at two fixed times, which are independent of the polynomial.
Also, show that these two times are 10 : 16 am and 1 : 44 pm to the nearest
minute.

(B3) A closed subset S of R2 lies in a < x < b. Show that its projection
on the y-axis is closed.

(B4) An automobile starts from rest and ends at rest, traversing a distance
of one mile in one minute, along a straight road. If a governor prevents the
speed of the car from exceeding 90 miles per hour, show that at some time of
the traverse the acceleration or deceleration of the car was at least 6.6 ft/sec.

(B5) Let un denote the ramp function

un (x) = −n f or x ≤ −n,

un (x) = x f or − n ≤ x ≤ n,

Page 66
Putnam In Olympiad Mahdiyar Amid Sardari

un (x) = n f or n ≤ x,
and let f be a real function of a real variable. Show that f is continuous if and
only if un ◦ f is continuous for all n.

(B6)
√ Show that if a circumscribe quadrilateral of sides a, b, c, d has area A =
abcd, then it is also indescribable.

1969
(A1) Let f (x, y) be a polynomial with real coefficients in the real variables x
and y defined over the entire xy-plane. What are the possibilities for the range
of f (x, y)?

(A3) Let P be a non-selfintersecting closed polygon with n sides. Let its vertices
be P1 , P2 , . . . , Pn . Let m other points,Q1 , Q2 , . . . , Qm , interior to P , be given.
Let the figure be triangulated. This means that certain pairs of the (n + m)
points P1 , . . . , Qm are connected by line segments such that
(1) the resulting figure consists exclusively of a set T of triangles,
(2) if two different triangles in T have more than a vertex in common then they
have exactly a side in common, and
(3) the set of vertices of the triangles in T is precisely the set of the (n + m)
points
P1 , . . . , Qm . How many triangles are in T ?

(A6) Let a sequence (xn ) be given and let yn = xn−1 + 2xn for n > 1. Suppose
that the sequence (yn ) converges. Prove that the sequence (xn ) converges, too.

(B1) Let n be a positive integer such that 24 | n + 1. Prove that the sum
of the positive divisors of n is divisible by 24.

(B2) Show that a finite group can not be the union of two of its proper sub-
groups. Does the statement remain true if ”two’ is replaced by ”three’ ?

(B3) The terms of a sequence (Tn ) satisfy Tn Tn+1 = n for all positive inte-
gers n and
Tn
lim = 1.
n→∞ Tn+1

Show that πT12 = 2.

(B4) Show that any curve of unit length can be covered by a closed rectan-
gle of area 1/4.

Page 67
Putnam In Olympiad Mahdiyar Amid Sardari

(B5) Let a1 < a2 < . . . be an increasing sequence of positive integers. Let


the series

X 1
a
i=1 i

be convergent. For any real number x, let k(x) be the number of the ai which
do not exceed x. Show that limx→∞ k(x)
x = 0.

1968
(A2) Given integers a, b, c, d, m, n such that ad − bc ̸= 0 and any real ε > 0,
show that one can find rational numbers x, y such that 0 < |ax + by − m| < ε
and 0 < |cx + dy − n| < ε.

(A3) Let S be a finite set and P the set of all subsets of S. Show that one
can label the elements of P as Ai such that
(1) A1 = ∅.
(2) For each n ≥ 1 we either have An−1 ⊂ An and |An \An−1 | = 1 or An ⊂ An−1
and |An−1 \ An | = 1.

(A4) Let S 2 ⊂ R3 be the unit sphere. Show that for any n points on S 2 ,
the sum of the squares of the n(n−1)
2 distances between them is at most n2 .

(A5) Find the smallest possible α ∈ R such that if P (x) = ax2 + bx + c satisfies
|P (x)| ≤ 1 for x ∈ [0, 1] , then we also have |P ′ (0)| ≤ α.

(A6) Find all polynomials whose coefficients are all ±1 and whose roots are
all real.

(B1) The random variables X, Y can each take a finite number of integer values.
They are not necessarily independent. Express P (min(X, Y ) = k) in terms of
p1 = P (X = k), p2 = P (Y = k) and p3 = P (max(X, Y ) = k).

(B2) Let G be a finite group with n elements and K a subset of G with more
than n2 elements. Show that for any g ∈ G one can find h, k ∈ K such that
g = h · k.

(B3) Given that a 60◦ angle cannot be trisected with ruler and compass, prove

that a 120
n angle cannot be trisected with ruler and compass for n = 1, 2, . . .

(B6) Show that one cannot find compact sets A1 , A2 , A3 , . . . in R such that
(1) All elements of An are rational.
(2) Any compact set K ⊂ R which only contains rational numbers is contained

Page 68
Putnam In Olympiad Mahdiyar Amid Sardari

in some Am .

1967
(A1) Let f (x) = a1 sin x + a2 sin 2x + · · · + an sin nx, where a1 , a2 , . . . , an are
real numbers and where n is a positive integer. Given that |f (x)| ≤ | sin x| for
all real x, prove that
|a1 + 2a2 + · · · + nan | ≤ 1.

(A3) Consider polynomial functions ax2 − bx + c with integer coefficients which


have two distinct zeros in the open interval (0, 1). Exhibit with proof the least
positive integer value of a for which such a polynomial exists.

(A5) Show that in a convex region in the plane whose boundary contains at
most a finite number of straight line segments and whose area is greater than
π
4 there is at least one pair of points a unit distance apart.

(A6) Given real numbers (ai ) and (bi ) (for i = 1, 2, 3, 4) such that a1 b2 ̸= a2 b1 .
Consider the set of all solutions (x1 , x2 , x3 , x4 ) of the simultaneous equations

a1 x1 + a2 x2 + a3 x3 + a4 x4 = 0 and b1 x1 + b2 x2 + b3 x3 + b4 x4 = 0

for which no xi is zero. Each such solution generates a 4-tuple of plus and minus
signs (by considering the sign of xi ).
(1) Determine, with proof, the maximum number of distinct 4-tuples possible.
(2) Investigate necessary and sufficient conditions on (ai ) and (bi ) such that the
above maximum of distinct 4-tuples is attained.

(B1) Let ABCDEF be a hexagon inscribed in a circle of radius r. Show that if


AB = CD = EF = r, then the midpoints of BC, DE and F A are the vertices
of an equilateral triangle.

(B2) Let 0 ≤ p, r ≤ 1 and consider the identities

a) (px+(1−p)y)2 = ax2 +bxy+cy 2 , b) (px+(1−p)y)(rx+(1−r)y) = αx2 +βxy+γy 2 .

Show that
4 4
a) max(a, b, c) ≥ , b) max(α, β, γ) ≥ .
9 9

(B4) 1) A certain locker room contains n lockers numbered 1, 2, . . . , n and all are
originally locked. An attendant performs a sequence of operations T1 , T2 , . . . , Tn ,

Page 69
Putnam In Olympiad Mahdiyar Amid Sardari

whereby with the operation Tk the state of those lockers whose number is divis-
ible by k is swapped. After all n operations have been performed, it is observed
that all lockers whose number is a perfect square (and only those lockers) are
open. Prove this.
2) Investigate in a meaningful mathematical way a procedure or set of opera-
tions similar to those above which will produce the set of cubes, or the set of
numbers of the form 2m2 , or the set of numbers of the form m2 + 1, or some
nontrivial similar set of your own selection.

(B5) Show that the sum of the first n terms in the binomial expansion of (2−1)−n
is 12 , where n is a positive integer.

1966
(A1) Let f (n) be the sum of the first n terms of the sequence 0, 1, 1, 2, 2, 3, 3, 4, . . . ,
where the nth term is given by

an = n/2 if n even,

an = (n − 1)/2 if n is odd.
Show that if x and y are positive integers and x > y then xy = f (x+y)−f (x−y).

(A2) Let a, b, c be the lengths of the sides of a triangle, let p = (a + b + c)/2,


and r be the radius of the inscribed circle. Show that
1 1 1 1
+ + ≥ 2.
(p − a)2 (p − b)2 (p − c)2 r

(A3) Let 0 < x1 < 1 and xn+1 = xn (1 − xn ), n = 1, 2, 3, . . . . Show that

lim nxn = 1.
n→∞

(A4) Prove that after deleting the perfect squares from the list
√ of positive inte-

gers the number we find in the√nth position is equal to n + { n}, where { n}
denotes the integer closest to n.

(A5) Let C denote the family of continuous functions on the real axis. Let
T be a mapping of C into C which has the following properties:
1) T is linear, i.e. T (c1 ψ1 + c2 ψ2 ) = c1 T ψ1 + c2 T ψ2 for c1 and c2 real and ψ1
and ψ2 in C.
2) T is local, i.e. if ψ1 ≡ ψ2 in some interval I then also T ψ1 ≡ T ψ2 holds in I.

Page 70
Putnam In Olympiad Mahdiyar Amid Sardari

Show that T must necessarily be of the form T ψ(x) = f (x)ψ(x) where f (x) is
a suitable continuous function.

(A6) Justify the statement that


v s
u r

u q
t
3 = 1 + 2 1 + 3 1 + 4 1 + 5 1 + . . ..

(B1) Let a convex polygon P be contained in a square of side one. Show that
the sum of the sides of P is less than or equal to 4.

(B2) Prove that among any ten consecutive integers at least one is relatively
prime to each of the others.

(B3) Show that if the series



X 1
p
n=1 n

is convergent, where p1 , p2 , p3 , . . . , pn , . . . are positive real numbers, then the


series

X n2
pn
n=1
(p1 + p2 + · · · + pn )2
is also convergent.

(B4) Let 0 < a1 < a2 < · · · < amn+1 be mn + 1 integers. Prove that you
can select either m + 1 of them no one of which divides any other, or n + 1 of
them each dividing the following one.

(B5) Given n(≥ 3) distinct points in the plane, no three of which are on the
same straight line, prove that there exists a simple closed polygon with these
points as vertices.

(B6) Show that all the solutions of the differential equation y ′′ + ex y = 0 remain
bounded as x → ∞.

1965
(A1) Let ABC be a triangle with angle A < angle C < 90◦ < angle B. Consider
the bisectors of the external angles at A and B, each measured from the vertex
to the opposite side (extended). Suppose both of these line-segments are equal
to AB. Compute the angle A.

Page 71
Putnam In Olympiad Mahdiyar Amid Sardari

(A2) Show that, for any positive integer n,


[(n−1)/2]   2  
X n − 2r n 1 2n − 2
= ,
r=0
n r n n−1
n

where [x] means the greatest integer not exceeding x, and r is the binomial
n

coefficient ”n choose r”, with the convention 0 = 1.

(A3) Show that, for any sequence a1 , a2 , . . . of real numbers, the two condi-
tions
e(ia1 ) + e(ia2 ) + · · · + e(ian )
lim =α
n→∞ n
and
e(ia1 ) + e(ia2 ) + · · · + e(ian2 )
lim =α
n→∞ n2
are equivalent.

(A4) At a party, assume that no boy dances with every girl but each girl dances
with at least one boy. Prove that there are two couples gb and g ′ b′ which dance
whereas b does not dance with g ′ nor does g dance with b′ .

(A5) In how many ways can the integers from 1 to n be ordered subject to
the condition that, except for the first integer on the left, every integer differs
by 1 from some integer to the left of it?

(A6) In the plane with orthogonal Cartesian coordinates x and y, prove that
the line whose equation is ux + vy = 1 will be tangent to the cirve xm + y m = 1
(where m > 1) if and only if un + v n = 1 and m−1 + n−1 = 1.

(B2) In a round-robin tournament with n players P1 , P2 , . . ., Pn (where n > 1),


each player plays one game with each of the other players and the rules are such
that no ties can occur. Let wr and lr be the number of games won and lost,
respectively, by Pr . Show that
n
X n
X
wr2 = lr2 .
r=1 r=1

(B3) Prove that there are exactly three right-angled triangles whose sides are
integers while the area is numerically equal to twice the perimeter.

(B4) Consider the function


n n n 2
  
0 + 2 x + 4 x + ···
f (x, n) = n n n 2 ,
1 + 3 x + 5 x + ···

Page 72
Putnam In Olympiad Mahdiyar Amid Sardari

where n is a positive integer. Express f (x, n + 1) rationally in terms of f (x, n)


and x. Hence, or otherwise, evaluate limn→∞ f (x, n) for suitable fixed values of
x. (The symbols n represent the binomial coefficients.)

(B5) Consider collections of unordered pairs of V different objects a, b, c, . . .,


k. Three pairs such as ab, bc, ab are said to form a triangle. Prove that, if
4E ≤ V 2 , it is possible to choose E pairs so that no triangle is formed.

(B6) If A, B, C, D are four distinct points such that every circle through
A and B intersects (or coincides with) every circle through C and D, prove
that the four points are either collinear (all on one line) or concyclic (all on one
circle).

1964
(A1) Given 6 points in a plane, assume that each two of them are connected by
a segment. Let D be the length of the
√ longest, and d the length of the shortest
of these segments. Prove that Dd ≥ 3.

(A3) Let P1 , P2 , . . . be a sequence of distinct points which is dense in the inter-


val (0, 1). The points P1 , . . . , Pn−1 decompose the interval into n parts, and Pn
decomposes one of these into two parts. Let an and bn be the length of these
two intervals. Prove that

X
an bn (an + bn ) = 1/3.
n=1

(A4) Let pn be a bounded sequence of integers which satisfies the recursion


pn−1 + pn−2 + pn−3 pn−4
pn = .
pn−1 pn−2 + pn−3 + pn−4

Show that the sequence eventually becomes periodic.

(A5) Prove that there exists a constant K such that the following inequality
holds for any sequence of positive numbers a1 , a2 , a3 , . . . :
∞ ∞
X n X 1
≤K .
a + a2 + . . . + an
n=1 1
a
n=1 n

(A6) Let S be a finite subset of a straight line. Say that S has the repeated

Page 73
Putnam In Olympiad Mahdiyar Amid Sardari

distance property if every value of the distance between two points of S (except
the longest) occurs at least twice. Show that if S has the repeated distance
property then the ratio of any two distances between two points of S is rational.

(B1) Let uk be a sequence of integers, and let Vn be the number of those which
are less than or equal to n. Show that if

X 1
< ∞,
uk
k=1

then
Vn
lim = 0.
n→∞ n

(B2) Let S be a set of n > 0 elements, and let A1 , A2 , . . . Ak be a family of


distinct subsets such that any two have a non-empty intersection. Assume that
no other subset of S intersects all of the Ai . Prove that k = 2n−1 .

(B3) Let f : R → R be a continuous function with the following property: for all
α ∈ R>0 , the sequence (an )n∈N defined as an = f (nα) satisfies limn→∞ an = 0.
Is it necessarily true that limx→+∞ f (x) = 0?

(B4) Into how many regions do n great circles, no three of which meet at a
point, divide a sphere?

(B5) Let un denote the least common multiple of the first n terms of a strictly
increasing sequence of positive integers. Prove that the series

X 1
u
n=1 n

is convergent.

1963
(A1) 1) Show that a regular hexagon, six squares, and six equilateral triangles
can be assembled without overlapping to form a regular dodecagon.
2) Let P1 , P2 , . . . , P12 be the vertices of a regular dodecagon. Prove that the
three diagonals P1 P9 , P2 P11 and P4 P12 intersect.

(A2) Let f : N → N be a strictly increasing function such that f (2) = 2


and f (mn) = f (m)f (n) for every pair of relatively prime positive integers m
and n. Prove that f (n) = n for every positive integer n.

Page 74
Putnam In Olympiad Mahdiyar Amid Sardari

(A4) Let (an ) be a sequence of positive real numbers. Show that


 
1 + an+1
lim sup n −1 ≥1
n→∞ an
and prove that 1 cannot be replaced by any larger number.

(A6) Let U and V be any two distinct points on an ellipse, let M be the mid-
point of the chord U V , and let AB and CD be any two other chords through
M . If the line U V meets the line AC in the point P and the line BD in the
point Q, prove that M is the midpoint of the segment P Q.

(B1) For which integers a does x2 − x + a divide x13 + x + 90 ?

(B2) Let S be the set of all numbers of the form 2m 3n , where m and n are
integers. Is S dense in the set of positive real numbers?

(B3) Find every twice-differentiable function f : R → R that satisfies the func-


tional equation
f (x)2 − f (y)2 = f (x + y)f (x − y)
for all x, y ∈ R.

(B4) Let C be a closed plane curve that has a continuously turning tangent
and bounds a convex region. If T is a triangle inscribed in C with maximum
perimeter, show that the normal to C at each vertex of T bisects the angle of T
at that vertex. If a triangle T has the property just described, does it necessarily
have maximum perimeter? What is the situation if C is a circle?

(B5) Let (an ) be a sequence of real numbers satisfying the inequalities


0 ≤ ak ≤ 100an for n ≤ k ≤ 2n and n = 1, 2, . . . ,
and such that the series

X
an
n=0
converges. Prove that
lim nan = 0.
n→∞

(B6) Let E be a Euclidean space of at most three dimensions. If A is a nonempty


subset of E, define S(A) to be the set of points that lie on closed segments join-
ing pairs of points of A (a one-point set should be considered to be a special
case of a closed segment). For a given nonempty set A0 , define An = S(An−1 )
for n = 1, 2, . . . Prove that A2 = A3 = . . . .

Page 75
Putnam In Olympiad Mahdiyar Amid Sardari

1962
(A1) Consider 5 points in the plane, such that there are no 3 of them collinear.
Prove that there is a convex quadrilateral with vertices at 4 points.

(A2) Find every real-valued function f whose domain is an interval I (finite


or infinite) having 0 as a left-hand endpoint, such that forpevery positive x ∈ I
the average of f over the closed interval [0, x] is equal to f (0)f (x).

(A3) In a triangle ABC, let A′ be a point on the segment BC, B ′ be a point


on the segment CA and C ′ a point on the segment AB such that
AB ′ BC ′ CA′

= ′ = ′ = k,
BC CA AB
where k is a positive constant. Let △ be the triangle formed by the interesctions
of AA′ , BB ′ and CC ′ . Prove that the areas of △ and ABC are in the ratio

(k − 1)2
.
k2 + k + 1

(A5) Evaluate
n  
X n
k2 .
k
k=0

(A6) Let S be a set of rational numbers such that whenever a and b are mem-
bers of S, so are ab and a + b, and having the property that for every rational
number r exactly one of the following three statements is true:

r ∈ S, −r ∈ S, r = 0.

Prove that S is the set of all positive rational numbers.

(B1) Let x(n) = x(x − 1) · · · (x − n + 1) for n a positive integer and let x(0) = 1.
Prove that
n  
(n)
X n (k) (n−k)
(x + y) = x y .
k
k=0

(B2) Let S be the set of all subsets of the positive integers. Construct a function
f : R → S such that f (a) is a proper subset of f (b) whenever a < b.

(B3) Let S be a convex region in the euclidean plane containing the origin.
Assume that every ray from the origin has at least one point outside S. Prove
that S is bounded.

Page 76
Putnam In Olympiad Mahdiyar Amid Sardari

(B4) The euclidean plane is divided into regions by drawing a finite number
of circles. Show that it is possible to color each of these regions either red or
blue in such a way that no two adjacent regions have the same color.

(B5) Prove that for every integer n greater than 1 :


 n  n  n n
3n + 1 1 2
< + + ... + < 2.
2n + 2 n n n

(B6) Let
n
X
f (x) = ak sin kx + bk cos kx,
k=0

where ak and bk are constants. Show that if |f (x)| ≤ 1 for x ∈ [0, 2π] and there
exist 0 ≤ x1 < x2 < . . . < x2n < 2π with |f (xi )| = 1, then f (x) = cos(nx + a)
for some constant a.

1961
(A2) For a real-valued function f (x, y) of two positive real variables x and y,
define f to be linearly bounded if and only if there exists a positive number K
such that |f (x, y)| < K(x + y) for all positive x and y. Find necessary and suffi-
cient conditions on the real numbers α and β such that xα y β is linearly bounded.

(A4) Let Ω(n) be the number of prime factors of n. Define f (1) = 1 and
f (n) = (−1)Ω(n) . Furthermore, let
X
F (n) = f (d).
d|n

Prove that F (n) = 0, 1 for all positive integers n. For which integers n is
F (n) = 1?

(A5) Let Ω be a set of n points, where n > 2. Let Σ be a nonempty sub


collection of the 2n subsets of Ω that is closed with respect to the unions, in-
tersections and complements. If k is the number of elements of Σ, what are the
possible values of k?

(A6) Prove that p(x) = 1 + x + x2 + . . . + xn is reducible over F2 in case


n + 1 is composite. If n + 1 is prime, is p(x) irreducible over F2 ?

(B1) Let a1 , a2 , a3 , . . . be a sequence of positive real numbers, define sn =

Page 77
Putnam In Olympiad Mahdiyar Amid Sardari

a1 +a2 +...+an a−1 +a−1 +...+a−1


n and rn = 1 2
n
n
. Given that limn→∞ sn and limn→∞ rn
exist, prove that the product of these limits is not less than 1.

(B2) Let a and b be given positive real numbers, with a < b. If two points
are selected at random from a straight line segment of length b, what is the
probability that the distance between them is at least a?

(B3) Consider four points in the plane, no three of which are collinear, and
such that the circle through three of them does not pass through the fourth.
Prove that one of the four points can be selected having the property that it
lies inside the circle determined by the other three.

(B4) Let x1 , x2 , . . . , xn be real numbers in [0, 1]. Determine the maximum value
of the sum of the n(n−1) 2 terms:
X
|xi − xj |.
i<j

(B5) Let k be a positive integer, and n a positive integer greater than 2. Define

f1 (n) = n, f2 (n) = nf1 (n) , . . . , fj+1 (n) = nfj (n) .

Prove either part of the inequality

fk (n) < n!! · · ·! < fk+1 (n),

where the middle term has k factorial symbols.

(B7) Given a sequence (an ) of non-negative real numbers such that an+m ≤

an am for all pairs of positive integers m and n, prove that the sequence ( n an )
converges.

1960
(A1) Let n be a given positive integer. How many solutions are there in ordered
positive integer pairs (x, y) to the equation
xy
= n?
x+y

(A2) Show that if three points


√ are √
inside are closed square of unit side, then
some pair of them are within 6 − 2 units apart.

Page 78
Putnam In Olympiad Mahdiyar Amid Sardari

(A4) Given two points, P and Q, on the same side of a line L, the problem
is to find a third point R so that P R + RQ + RS is minimal, where S is the
unique point on L such that RS is perpendicular to L. Consider all cases.

(A5) Find all polynomials f (x) with real coefficients having the property f (g(x)) =
g(f (x)) for every polynomial g(x) with real coefficients.

(A6) A player repeatedly throwing a die is to play until their score reaches
or passes a total n. Denote by p(n) the probability of making exactly the total
n, and find the value of limn→∞ p(n).

(A7) Let N (n) denote the smallest positive integer N such that xN = e for
every element x of the symmetric group Sn , where e denotes the identity per-
mutation. Prove that if n > 1,
N (n)
= p if n a power of a prime p
N (n − 1)
N (n)
= 1 otherwise.
N (n − 1)

(B1) Find all integer solutions (m, n) to mn = nm .

(B2) Evaluate the double series


∞ X

X 2
2−3k−j−(k+j) .
j=0 k=0

(B4) Consider the arithmetic progression a, a + d, a + 2d, . . . where a and d


are positive integers. For any positive integer k, prove that the progression has
either no k-th powers or infinitely many.

(B5) efine a sequence (an ) by a0 = 0 and an = 1 + sin(an−1 − 1) for n ≥ 1.


Evaluate
n
1X
lim ak .
n→∞ n
k=1

1959
(A1) Let n be a positive integer. Prove that xn − x1n is expressible as a polyno-
mial in x − x1 with real coefficients if and only if n is odd.

Page 79
Putnam In Olympiad Mahdiyar Amid Sardari

(A2) Let ω 3 = 1, ω ̸= 1. Show thatz1 , z2 , −ωz1 , −ω 2 z2 are the vertices of an


equilateral triangle.

(A3) Find all complex-valued functions f of a complex variable such that

f (z) + zf (1 − z) = 1 + z

for all z ∈ C.

(A4) If f and g are real-valued functions of one real variable, show that there
exist x and y in [0, 1] such that
1
|xy − f (x) − g(y)| ≥ .
4

(A5) A sparrow, flying horizontally in a straight line, is 50 feet directly be-


low an eagle and 100 feet directly above a hawk. Both hawk and eagle fly
directly toward the sparrow, reaching it simultaneously. The hawk flies twice as
fast as the sparrow. How far does each bird fly? At what rate does the eagle fly?

(B1) Let each of m distinct points on the positive part of the x-axis be joined
to n distinct points on the positive part of the y-axis. Obtain a formula for the
number of intersection points of these segments, assuming that no three of the
segments are concurrent.

(B2) Let c be a positive real number. Prove that c can be expressed in in-
finitely many ways as a sum of infinitely many distinct terms selected from the
1

sequence 10n n∈N

(B3) Give an example of a continuous real-valued function f form [0, 1] to [0, 1]


which takes on every value in [0, 1] an infinite number of times.

(B6) Let α and β be irrational numbers with the property that


1 1
+ =1
α β
Let{an } and {bn } be the sequences given by an = ⌊nα⌋ and bn = ⌊nβ⌋ respec-
tively. Prove that the sequences {an } and {bn } has no term in common and
cover all the natural numbers.

(B7) For each positive integer n, let fn be a real-valued symmetric function of


n real variables. Suppose that for all n and all real numbers x1 , . . . , xn , xn+1 , y
it is true that
(1) fn (x1 + y, . . . , xn + y) = fn (x1 , . . . , xn ) + y,

Page 80
Putnam In Olympiad Mahdiyar Amid Sardari

(2) fn (−x1 , . . . , −xn ) = −fn (x1 , . . . , xn ),


(3) fn+1 (fn (x1 , . . . , xn ), . . . , fn (x1 , . . . , xn ), xn+1 ) = fn+1 (x1 , . . . , xn ).
Prove that fn (x1 , . . . , xn ) = x1 +···+x n
n
.

2
1958-F
(A1) If a0 , a1 , . . . , an are real number satisfying
a0 a1 an
+ + ... + = 0,
1 2 n+1
show that the equation an xn + . . . + a1 x + a0 = 0 has at least one real root.

(A2) Two uniform solid spheres of equal radii are so placed that one is di-
rectly above the other. The bottom sphere is fixed, and the top sphere, initially
at rest, rolls off. At what point will contact between the two spheres be ”lost”?
Assume the coefficient of friction is such that no slipping occurs.

(A3) Real numbers are chosen at random from the interval [0, 1]. If after choos-
ing the n-th number the sum of the numbers so chosen first exceeds 1, show
that the expected value for n is e.

(A4) If a1 , a2 , . . . , an are complex numbers such that

|a1 | = |a2 | = · · · = |an | = r ̸= 0,

and if Ts denotes the sum of all products of these n numbers taken s at a time,
prove that
Ts 2s−n
Tn−s = r

whenever the denominator of the left-hand side is different from 0.

(A6) What is the smallest amount that may be invested at interest rate i,
compounded annually, in order that one may withdraw 1 dollar at the end of
the first year, 4 dollars at the end of the second year, . . . , n2 dollars at the end
of the n-th year, in perpetuity?

(A7) Show that ten equal-sized squares cannot be placed on a plane in such
a way that no two have an interior point in common and the first touches each
of the others.

(B2) Prove that the product of four consecutive positive integers cannot be
a perfect square or cube.
2 In 1958 Putnam held twice in a year in February and November.

Page 81
Putnam In Olympiad Mahdiyar Amid Sardari

(B3) In a round-robin tournament with n players in which there are no draws,


the numbers of wins scored by the players are s1 , s2 , . . . , sn . Prove that a neces-
sary and sufficient condition for the existence of three players A, B, C such that
A beats B, B beats C, and C beats A is

(2n − 1)(n − 1)n


s21 + s22 + . . . + s2n < .
6

(B4) Title is self explanatory. Pick two points on the unit sphere. What is
the expected distance between them?

(B5) S is an infinite set of points in the plane. The distance between any
two points of S is integral. Prove that S is a subset of a straight line.

1958-N
(A1) Let f (m, 1) = f (1, n) = 1 for m ≥ 1, n ≥ 1 and let f (m, n) = f (m − 1, n) +
f (m, n − 1) + f (m − 1, n − 1) for m > 1 and n > 1. Also let
X
S(n) = f (a, b) a ≥ 1 and b ≥ 1.
a+b=n

Prove that
S(n + 2) = S(n) + 2S(n + 1) for n ≥ 2.

(A2) Let R1 = 1 and Rn+1 = 1 + n/Rn for n ≥ 1. Show that for n ≥ 1,


√ √
n ≤ Rn ≤ n + 1.

(A4) In assigning dormitory rooms, a college gives preference to pairs of students


in this order:

AA, AB, AC, BB, BC, AD, CC, BD, CD, DD

in which AA means two seniors, AB means a senior and a junior, etc. Deter-
mine numerical values to assign to A, B, C and D so that the set of numbers
A + A, A + B, A + C, B + B, . . . corresponding to the order above will be in
descending order. Find the general solution and the solution in least positive
integers.

Page 82
Putnam In Olympiad Mahdiyar Amid Sardari

(A5) Show that the number of non-zero integers in the expansion of the n-
th order determinant having zeroes in the main diagonal and ones elsewhere
is
(−1)n
 
1 1 1
n! 1 − + − + · · · + .
1! 2! 3! n!

(A6) Let a(x) and b(x) be continuous functions on [0, 1] and let 0 ≤ a(x) ≤ a < 1
on that range. Under what other conditions (if any) is the solution of the equa-
tion for u,
u = max b(x) + a(x)u
0≤x≤1

given by
b(x)
u = max .
0≤x≤1 1 − a(x)

(A7) Let a and b be relatively prime positive integers, b even. For each positive
integer q, let p = p(q) be chosen so that

p a

q b

is a minimum. Prove that



n q p − a
X q b 1
lim = .
n→∞
q=1
n 4

(B1) Given
n  −1
X n
bn = , n ≥ 1,
k
k=0

prove that
n+1
bn = bn−1 + 1, n ≥ 2.
2n
Hence, as a corollary, show
lim bn = 2.
n→∞

(B2) Among any 2n − 1 integers, there are some n whose sum is divisible by n

(B3) Show that if a unit square is partitioned into two sets, then the diam-
eter (least upper bound of the distances between pairs of points) of one of the

Page 83
Putnam In Olympiad Mahdiyar Amid Sardari


sets is not less than 5/2. Show also that no larger number will do.

(B5) The lengths of successive segments of a broken line are represented by


the successive terms of the harmonic progression 1, 1/2, 1/3, . . . . Each segment
makes with the preceding a given angle θ. What is the distance and what is the
direction of the limiting points (if there is one) from the initial point of the first
segment?

(B6) Let a complete oriented graph on n points be given. Show that the vertices
can be enumerated as v1 , v2 , . . . , vn such that v1 → v2 → · · · → vn .

(B7) Let a1 , a2 , . . . , an be a permutation of the integers 1, 2, . . . , n. Call ai a


big integer if ai > aj for all i < j. Find the mean number of big integers over
all permutations on the first n positive integers.

1957
(A1) The normals to a surface all intersect a fixed straight line. Show that the
surface is a portion of a surface of revolution.

(A3) Let a, b be real numbers and k a positive integer. Show that



cos kb cos a − cos ka cos b
< k2 − 1
cos b − cos a

whenever the left side is defined.

(A5) Given n points in the plane, show that the largest distance determined
by these points cannot occur more than n times.

(A7) Each member of a set of circles in the xy-plane is tangent to the x-axis
and no two of the circles intersect. Show that
(1) the points of tangency can include all rational points on the axis.
(2) the points of tangency cannot include all the irrational points.

(B2) In order to determine A1 for A > 0, one can use the iteration Xk+1 =
Xk (2 − AXk ), where X0 is a selected starting value. Find the limitation, if any,
on the starting value X0 so that the above iteration converges to A1 .

(B4) Let a(n) be the number of representations of the positive integer n as


an ordered sum of 1’s and 2’s. Let b(n) be the number of representations of
the positive integer n as an ordered sum of integers greater than 1. Show that
a(n) = b(n + 2) for each n.

Page 84
Putnam In Olympiad Mahdiyar Amid Sardari

(B5) Let f be an increasing mapping from the family of subsets of a given finite
set H into itself, i.e. such that for every X ⊆ Y ⊆ H we have f (X) ⊆ f (Y ) ⊆ H.
Prove that there exists a subset H0 of H such that f (H0 ) = H0 .

(B7) Let C consist of a regular polygon and its interior. Show that for each
positive integer n, there exists a set of points S(n) in the plane such that every
n points can be covered by C, but S(n) cannot be covered by C.

1956
(A2) Prove that every positive integer has a multiple whose decimal represen-
tation involves all ten digits.

(A5) Call a subset of {1, 2, . . . , n} unfriendly if no two of its elements are consec-
utive. Show that the number of unfriendly subsets with k elements is n−k+1

k .

(A6) 1) A transformation of the plane into itself preserves all rational distances.
Prove that it preserves all distances.
2) Show that the corresponding statement for the line is false.

(A7) Prove that the number of odd binomial coefficients in any finite bino-
mial expansion is a power of 2.

(B2) Suppose that each set X of points in the plane has an associated set
X of points called its cover. Suppose further that (1) X ∪ Y ⊃ X ∪ Y ∪ Y for
all sets X, Y . Show that i) X ⊃ X, ii) X = X and iii) X ⊃ Y ⇒ X ⊃ Y . Prove
also that these three statements imply (1).

(B3) A sphere is inscribed in a tetrahedron and each point of contact of the


sphere with the four faces is joined to the vertices of the face containing the
point. Show that the four sets of three angles so formed are identical.

(B4) Prove that if A, B, and C are angles of a triangle measured in radians


then A cos B + sin A cos C > 0.

(B5) Show that a graph with 2n points and n2 + 1 edges necessarily contains a
3 − cycle, but that we can find a graph with 2n points and n2 edges without a
3 − cycle.
please prove it without induction .

(B6) Given T1 = 2, Tn+1 = Tn2 − Tn + 1 for n > 0. Prove:


(1) If
Pm ̸= n, Tm and Tn have no common factor greater than 1.

(2) i=1 T1i = 1.

Page 85
Putnam In Olympiad Mahdiyar Amid Sardari

(B7) The polynomials P (z) and Q(z) with complex coefficients have the same
set of numbers for their zeros but possibly different multiplicities. The same is
true for the polynomials

P (z) + 1 and Q(z) + 1.

Prove that

1955
(A1) Prove that there is no set of integers m, n, p except 0, 0, 0 for which
√ √
m + n 2 + p 3 = 0.

(A2) A1 A2 . . . An is a regular polygon inscribed in a circle of radius r and


center O. P is a point on line OA1 extended beyond A1 . Show that:
n
Y n
P A i = OP − rn .
i=1

P∞
(A3) Suppose that i=1 xi is a convergent series of positive terms which mono-
tonically decrease (that is, x1 ≥ x2 ≥ x3 ≥ · · · ). Let P denotePthe set of all

numbers which are sums of some (finite or infinite) subseries of i=1 xi . Show
that P is an interval if and only if:

X
xn ≤ xi
i=n+1

for every integer n.

(A4) On a circle, n points are selected and the chords joining them in pairs
are drawn. Assuming that no three of these chords are concurrent (except at
the endpoints), how many points of intersection are there?

(A5) If a parabola is given in the plane, find a geometric construction (ruler


and compass) for the focus.

(A6) Find a necessary and sufficient condition on the positive integer n that
the equation:
xn + (2 + x)n + (2 − x)n = 0

Page 86
Putnam In Olympiad Mahdiyar Amid Sardari

have a rational root.

(B1) A sphere rolls along two intersecting straight lines. Find the locus of
its center.

(B3) Prove that there exists no distance-preserving map of a spherical cap into
the plane. (Distances on the sphere are to be measured along great circles on
the surface.)

(B4) Do there exist 1, 000, 000 consecutive integers each of which contains a
repeated prime factor?

(B5) Given an infinite sequence of 0′ s and 1′ s and a fixed integer k, suppose


that there are no more than k distinct blocks of k consecutive terms. Show that
the sequence is eventually periodic.

(B6) Prove: If f (x) > 0 for all x and f (x) → 0 as x → ∞, then there ex-
ists at most a finite number of: solutions of

f (m) + f (n) + f (p) = 1

in positive integers m, n, and p.

(B7) Four forces acting on a body are in equilibrium. Prove that, if their lines
of action are mutually skew, they are rulings of a hyperboloid.

1954
(A1) Let n be an odd integer greater than 1. Let A be an n × n symmetric ma-
trix such that each row and column consists of some permutation of the integers
1, 2, . . . , n. Show that each of the integers 1, 2, . . . , n must appear in the main
diagonal of A.

(A2) Consider any five points in the interior of square S of side length
√ 1. Prove
that at least one of the distances between these points is less than 2/2. Can
this constant be replaced by a smaller number?

(A4) A uniform rod of length 2k and weight w rests with the end A against
a vertical wall, while the lower end B is fastened by a string BC of length 2b
coming from a point C in the wall above A. If the system is in equilibrium,
determine the angle ABC.

Page 87
Putnam In Olympiad Mahdiyar Amid Sardari

(A5) Let f (x) be a real-valued function defined for 0 < x < 1. If


x
lim f (x) = 0 and f (x) − f = o(x),
x→0 2
prove that f (x) = o(x), where we use the O-notation.

(A6) Suppose that u0 , u1 , . . . is a sequence of real numbers such that:



X
un = u2n+k for n = 0, 1, 2, . . .
k=1
P
Prove that if un converges, then uk = 0 for all k.

(A7) Prove that there are no integers x and y for which:

x2 + 3xy − 2y 2 = 122.

(B1) Show that the equation x2 − y 2 = a3 has always integral solutions for
x and y whenever a is a positive integer.

(B2) Let s denote the sum of the alternating harmonic series. Rearrange this
series as follows:
1 1 1 1 1 1 1
1+ − + + − + + − ...
3 2 5 7 4 9 11
Assume as known that this series converges as well and denote its sum by S.
Denote by sk , Sk respectively the k-th partial sums of both series. Prove that
1) S3n = s4n + 12 s2n .
2) S ̸= s.

(B3) Let [a1 , b1 ], . . . , [an , bn ] be a collection of closed intervals such that any
of these closed intervals have a point in common. Prove that there exists a
point contained in every one of these intervals.

(B4) Given the focus F and the directrix D of a parabola P and a line L,
describe a euclidean construction for the point or points of intersection of P
and L. Be sure to identify the case for which there are no points of intersection.

(B6) Let: x ∈ Q+ . Prove that there exits α1 , α2 , ..., αk ∈ N and pairwe distinct
such that
k
X 1
x= .
α
i=1 i

Page 88
Putnam In Olympiad Mahdiyar Amid Sardari

1953
(A1) Prove that for every positive integer n
2 √ √ √ √ 4n + 3 √
n n < 1 + 2 + ... + n < n.
3 6

(A2) The complete graph with 6 points and 15 edges has each edge colored
red or blue. Show that we can find 3 points such that the 3 edges joining them
are the same color.

(A3) a, b, c are real, and the sum of any two is greater than the third. Show
that:
2(a + b + c)(a2 + b2 + c2 )
> a3 + b3 + c3 + abc.
3

(A5) S is a parabola with focus F and axis L. Three distinct normals to S


pass through P. Show that the sum of the angles which these make with L less
the angle which P F makes with L is a multiple of π.

(A6) Show that the sequence:


r
√ √ √
q q
7, 7− 7, 7− 7− 7, . . .

converges and evaluate the limit.

(A7) Assuming that the roots of x3 + px2 + qx + r = 0 are all real and positive,
find the relation between p, q, r which is a necessary and sufficient condition
that the roots are the cosines of the angles of a triangle.

(B2) Let a0 , a1 , . . . , an be real numbers and let f (x) = an xn + . . . + a1 x + a0 .


Suppose that f (i) is an integer for all i. Prove that n!·ak is an integer for each k.

(B4) Determine the equations of a surface in three-dimensional Cartesian space


which has the following properties:
(1) it passes through the point (1, 1, 1) and
(2) if the tangent plane is drawn at any point P and X, Y, Z are the intersections
of this plane with the x, y and z−axis respectively, then P is the orthocenter of
the triangle XY Z.

(B5) how that the roots of x4 + ax3 + bx2 + cx + d, if suitably numbered,


satisfy the relation rr21 = rr34 , provided a2 d = c2 ̸= 0.

(B6) Let P and Q be any points inside a circle C with center O such that

Page 89
Putnam In Olympiad Mahdiyar Amid Sardari

OP = OQ. Determine the location of a point Z on C such that P Z + QZ is


minimal.

(B7) Let w ∈ (0, 1) be an irrational number. Prove that w has a unique conver-
gent expansion of the form:
1 1 1 1
w= − + − + ...,
p0 p0 p1 p0 p1 p2 p0 p1 p2 p3

where 1 ≤ p0 < p1 < p2 < . . . are integers. If w = √1 , find p0 , p1 , p2 .


2

1952
(A1) Let:
i=n
X
f (x) = ai xn−i
i=0

be a polynomial of degree n with integral coefficients. If a0 , an , and f (1) are


odd, prove that f (x) = 0 has no rational roots.

(A3) Develop necessary and sufficient conditions which ensure that r1 , r2 , r3


and r12 , r22 , r32 are simultaneously roots of the equation x3 + ax2 + bx + c = 0.

(A4) The flag of the United Nations consists of a polar map of the world, with
the North Pole as its center, extending to approximately 45◦ South Latitude.
The parallels of latitude are concentric circles with radii proportional to their
co-latitudes. Australia is near the periphery of the map and is intersected by
the parallel of latitude 30◦ S.In the very close vicinity of this parallel how much
are East and West distances exaggerated as compared to North and South dis-
tances?

(A5) Let aj (j = 1, 2, . . . , n) be entirely arbitrary numbers except that no one is


equal to unity. Prove:
n
X i−1
Y n
Y
a1 + ai (1 − aj ) = 1 − (1 − aj ).
i=2 j=1 j=1

(A6) A man has a rectangular block of wood m by n by r inches (m, n, and r


are integers). He paints the entire surface of the block, cuts the block into inch
cubes, and notices that exactly half the cubes are completely unpainted. Prove
that the number of essentially different blocks with this property is finite. (Do
not attempt to enumerate them.)

Page 90
Putnam In Olympiad Mahdiyar Amid Sardari

(A7) Directed lines are drawn from the center of a circle, making angles of
0, ±1, ±2, ±3, . . . (measured in radians from a prime direction). If these lines
meet the circle in points P0 , P1 , P−1 , P2 , P−2 , . . . , show that there is no interval
on the circumference of the circle which does not contain some P±i . (You may
assume that π is irrational.)

(B4) A homogeneous solid body is made by joining a base of a circular cylinder


of height h and radius r, and the base of a hemisphere of radius r. This body
is placed with the hemispherical end on a horizontal table, with the axis of the
cylinder in a vertical position, and then slightly oscillated. It is intuitively ev-
ident that if r is large as compared to h, the equilibrium will be stable; but if
r is small compared to h, the equilibrium will be unstable. What is the critical
value of the ratio r/h which enables the body to rest in neutral equilibrium in
any position?
P∞
(B5) If the terms P
of a sequence a1 , a2 , . . . are monotonic, and if n=1 an con-

verges, show that n=1 n(an − an+1 ) converges.

(B6) Prove the necessary and sufficient condition that a triangle inscribed in an
ellipse shall have maximum area is that its centroid coincides with the center of
the ellipse.

(B7) Given any real number N0 , if Nj+1 = cos Nj , prove that limj→∞ Nj exists
and is independent of N0 .

1951
(A2) In the plane, what is the locus of points of the sum of the squares of whose
distances from n fixed points is a constant? What restrictions, stated in geo-
metric terms, must be put on the constant so that the locus is non-null?

(A3) Find the sum to infinity of the series:

1 1 1 (−1)n+1
1− + − + ··· + + ··· .
4 7 10 3n − 2

(A4) Trace the curve whose equation is:

y 4 − x4 − 96y 2 + 100x2 = 0.

Page 91
Putnam In Olympiad Mahdiyar Amid Sardari

(A5) Consider in the plane the network of points having integral coordinates.
For lines having rational slope show that:
(1) the line passes through no points of the network or through infinitely many;
(2) there exists for each line a positive number d having the property that no
point of the network, except such as may be on the line, is closer to the line
than the distance d.

(A6) Determine the position of a normal chord of a parabola such that it cuts
off of the parabola a segment of minimum area.

(A7) Show that if the series a1 + a2 + a3 + · · · + an + · · · converges, then


the series a1 + a2 /2 + a3 /3 + · · · + an /n + · · · converges also.

(B4) Investigate, in any way which yields significant results, the existence, in
the plane, of the configuration consisting of an ellipse simultaneously tangent
to four distinct concentric circles.

(B5) A plane through the center of a torus is tangent to the torus. Prove
that the intersection of the plane and the torus consists of two circles.

(B6) Assuming that all of the roots of the cubic equation x3 + ax2 + bx + c = 0
are real, show that the difference between the greatest and the least roots is not
less than (a2 − 3b)1/2 or greater than 2(a2 − 3b)1/2 /31/2 .

(B7) Find the volume of the four-dimensional hypersphere x2 + y 2 + z 2 + t2 = r2


and the hypervolume of its interior x2 + y 2 + z 2 + t2 < r2

1950
(A1) For what values of the ratio a/b is the limaçon r = a − b cos θ a convex
curve? (a > b > 0).

(A3) The sequence x0 , x1 , x2 , . . . is defined by the conditions


xn−1 + (2n − 1) xn
x0 = a, x1 = b, xn+1 =
2n
for n ≥ 1, where a and b are given numbers. Express limn→∞ xn concisely in
terms of a and b.

(A6) Each coefficient an of the power series


a0 + a1 x + a2 x2 + a3 x3 + · · · = f (x)
has either the value of 1 or the value 0. Prove the easier of the two assertions:
(1) If f (0.5) is a rational number, f (x) is a rational function.

Page 92
Putnam In Olympiad Mahdiyar Amid Sardari

(2) If f (0.5) is not a rational number, f (x) is not a rational function.

(B1) In each of n houses on a straight street are one or more boys. At what
point should all the boys meet so that the sum of the distances that they walk
is as small as possible?

(B2) Two obvious approximations to the length of the perimeter of the ellipse
with semi-axes a and b are π(a + b) and 2π(ab)1/2 . Which one comes nearer the
truth when the ratio b/a is very close to 1?

(B3) In the Gregorian calendar:


(1) years not divisible by 4 are common years;
(2) years divisible by 4 but not by 100 are leap years;
(3) years divisible by 100 but not by 400 are common years;
(4) years divisible by 400 are leap years;
(5) a leap year contains 366 days; a common year 365 days.
Prove that the probability that Christmas falls on a Wednesday is not 1/7.

(B4) The cross-section of a right cylinder is an ellipse, with semi-axes a and


b, where a > b. The cylinder is very long, made of very light homogeneous
material. The cylinder rests on the horizontal ground which it touches along
the straight line joining the lower endpoints of the minor axes of its several
cross-sections. Along the upper endpoints of these minor axes lies a very heavy
homogeneous wire, straight and just as long as the cylinder. The wire and
the cylinder are rigidly connected. We neglect the weight of the cylinder, the
breadth of the wire, and the friction of the ground.
The system described is in equilibrium, because of its symmetry. This equilib-
rium seems to be stable when the ratio b/a is very small, but unstable when this
ratio comes close to 1. Examine this assertion and find the value of the ratio
b/a which separates the cases of stable and unstable equilibrium.

(B5) Answer either (1) or (2).


(1) Given that the sequence whose nth term is (sn + 2sn+1 ) converges, show
that the sequence {sn } converges also.
(2) A plane varies so that it includes a cone of constant value equal to πa3 /3
with the surface the equation of which in rectangular coordinates is 2xy = z 2 .
Find the equation of the envelope of the various positions of this plane.
State the result so that it applies to a general cone (that is, conic surface) of
the second order.

(B6) Consider the closed plane curves Ci and Co , their respective lengths |Ci |
and |Co |, the closed surfaces Si and So , and their respective areas |Si | and |So |.
Assume that Ci lies inside Co and Si inside So . (Subscript i stands for ”inner,”
o for ”outer.”) Prove the correct assertions among the following four, and dis-
prove the others.
(1) If Ci is convex, |Ci | ≤ |Co |.

Page 93
Putnam In Olympiad Mahdiyar Amid Sardari

(2) If Si is convex, |Si | ≤ |So |.


(3) If Co is the smallest convex curve containing Ci , then |Co | ≤ |Ci |.
(4) If So is the smallest convex surface containing Si , then |So | ≤ |Si |.
You may assume that Ci and Co are polygons and Si and So polyhedra.

1949
(A1) Answer either (1) or (2):
(1) Let a > 0. Three straight lines pass through the three points (0, −a, a), (a, 0, −a)
and (−a, a, 0), parallel to the x−, y− and z−axis, respectively. A variable
straight line moves so that it has one point in common with each of the three
given lines. Find the equation of the surface described by the variable line.
(2) Which planes cut the surface xy + yz + xz = 0 in (A) circles, (B) parabolas?

(A2) We consider three vectors drawn from the same initial point O, of lengths
a, b and c, respectively. Let E be the parallelepiped with vertex O of which the
given vectors are the edges and H the parallelepiped with vertex O of which
the given vectors are the altitudes. Show that the product of the volumes of E
and H equals (abc)2 and generalize this result to n dimensions.

(A3) Assume that the complex numbers a1 , a2 , . . . are all different from 0, and
that |ar − as | > 1 for r ̸= s. Show that the series

X 1
a3
n=1 n

converges.

(A4) Given that P is a point inside a tetrahedron with vertices at A, B, C


and D, such that the sum of the distances P A + P B + P C + P D is a minimum,
show that the two angles ̸ AP B and ̸ CP D are equal and are bisected by the
same straight line. What other pair of angles must be equal?

(A5) How many roots of the equation z 6 + 6z + 10 = 0 lie in each quadrant of


the complex plane?
p
(B1) Each rational number with p, q coprime of the open interval (0, 1) is
q
h i √
covered by the closed interval pq − q12 , pq + q12 . Prove that 22 is not covered
by any of the above closed intervals.

(B3) Let K be a closed plane curve such that the distance between any two
points of K is always less than 1. Show that K lies in a circle of radius √13 .

Page 94
Putnam In Olympiad Mahdiyar Amid Sardari

(B4) Show that the coefficients a1 , a2 , a3 , . . . in the expansion


 
1 1
1+x− √ = a1 x + a2 x2 + a3 x3 + . . .
4 1 − 6x + x2
are positive integers.

(B5) Let (an ) be an arbitrary sequence of positive numbers. Show that:


 n
a1 + an+1
lim sup ≥ e.
n→∞ an

(B6) Let C be a closed convex curve with a continuously turning tangent and
let O be a point inside C. For each point P on C we define T (P ) as follows:
Draw the tangent to C at P and from O drop the perpendicular to that tangent.
Then T (P ) is the point at which C intersects this perpendicular. Starting now
with a point P0 on C, define points Pn by Pn = T (Pn−1 ). Prove that the points
Pn approach a limit and characterize all possible limit points. (You may assume
that T is continuous.)

1948
(A1) What is the maximum of |z 3 − z + 2|, where z is a complex number with
|z| = 1?

(A2) Two spheres in contact have a common tangent cone. These three surfaces
divide the space into various parts, only one of which is bounded by all three
surfaces, it is ”ring-shaped.” Being given the radii of the spheres, r and R, find
the volume of the ”ring-shaped” part.

(A3) Let (an ) be a decreasing sequence of positive numbers with limit 0 such
that:
bn = an − 2an+1 + an+2 ≥ 0
for all n. Prove that:

X
nbn = a1 .
n=1

(A6) Answer either (1) or (2):


(1) A force acts on the element ds of a closed plane curve. The magnitude of
this force is r−1 ds where r is the radius of curvature at the point considered,
and the direction of the force is perpendicular to the curve, it points to the

Page 95
Putnam In Olympiad Mahdiyar Amid Sardari

convex side. Show that the system of such forces acting on all elements of the
curve keep it in equilibrium.
(2) Show that:
2 2·4 5 2·4·6 7 arcsin x
x + x3 + x + x + ... = √ .
3 3·5 3·5·7 1 − x2

(B2) A circle moves so that it is continually in the contact with all three coor-
dinate planes of an ordinary rectangular system. Find the locus of the center of
the circle.
√ √ √
(B3) Prove that [ n + n + 1] = [ 4n + 1] for all n ∈ N .

(B5) The pairs (a, b) such that |a + bt + t2 | ≤ 1 for 0 ≤ t ≤ 1 fill a certain


region in the plane. What is the area of this region?

1947
(A1) If (an ) is a sequence of real numbers such that for n ≥ 1

(2 − an )an+1 = 1,

prove that: limn→∞ an = 1.

(A2) A real valued continuous function f satisfies for all real x and y the func-
tional equation p
f ( x2 + y 2 ) = f (x)f (y).
Prove that 2
f (x) = f (1)x .

(A3) Given a triangle ABC with an interior point P and points Q1 , Q2 not
lying on any of the segments AB, AC, BC, AP, BP, CP, show that there does
not exist a polygonal line K joining Q1 and Q2 such that
1) K crosses each segment exactly once,
2) K does not intersect itself
3) K does not pass through A, B, C or P.

(A5) Let a1 , b1 , c1 be positive real numbers whose sum is 1, and for n = 1, 2, . . .


we define:

an+1 = a2n + 2bn cn , bn+1 = b2n + 2an cn , cn+1 = c2n + 2an bn .

Page 96
Putnam In Olympiad Mahdiyar Amid Sardari

Show that an , bn , cn approach limits as n → ∞ and find those limits.

(B3) Let x, y be Cartesian coordinates in the plane. I denotes the line seg-
ment 1 ≤ x ≤ 3, y = 1. For every point P on I, let P ′ denote the point that lies
on the segment joining the origin to P and such that the distance P P ′ is equal
to 1/100. As P describes I, the point P ′ describes a curve C. Which of I and
C has greater length?

(B4) Given P (z) = z 2 + az + b, where a, b ∈ C. Suppose that |P (z)| = 1


for every complex number z with |z| = 1. Prove that a = b = 0.

(B5) Let a, b, c, d be distinct integers such that:

(x − a)(x − b)(x − c)(x − d) − 4 = 0

has an integer root r. Show that 4r = a + b + c + d.

(B6) Let OX, OY, OZ be mutually orthogonal lines in space. Let C be a fixed
point on OZ and U, V variable points on OX, OY, respectively. Find the locus
of a point P such that P U, P V, P C are mutually orthogonal.

3
1946
(A3) A projectile in flight is observed simultaneously from four radio stations
which are situated at the corners of a square of side b. The distances of the
projectile from the four stations, taken in order around the square, are found
to be R1 , R2 , R3 and R4 . Show that:

R12 + R32 = R22 + R42 .

Show also that the height h of the projectile above the ground is given by
1 1 1
h2 = − b2 + (R12 + R22 + R32 + R42 ) − 2 (R14 + R24 + R34 + R44 − 2R12 R32 − 2R22 R42 ).
2 4 8b

(A5) Find the smallest volume bounded by the coordinate planes and by a
tangent plane to the ellipsoid:

x2 y2 z2
2
+ 2 + 2 = 1.
a b c

3 Because of the WWII, Putnam didn’t held in 1943-1945.

Page 97
Putnam In Olympiad Mahdiyar Amid Sardari

(B2) Let A, B be two variable points on a parabola P0 , such that the tan-
gents at A and B are perpendicular to each other. Show that the locus of the
centroid of the triangle formed by A, B and the vertex of P0 is a parabola P1 .
Apply the same process to P1 and repeat the process, obtaining the sequence
of parabolas P1 , P2 , . . . , Pn . If the equation of P0 is y = mx2 , find the equation
of Pn .

(B3) In a solid sphere of radius R the density ρ is a function of r, the dis-


tance from the center of the sphere. If the magnitude of the gravitational force
of attraction due to the sphere at any point inside the sphere is kr2 , where k
is a constant, find ρ as a function of r. Find also the magnitude of the force of
attraction at a point outside the sphere at a distance r from the center.

(B4) For each positive integer n, put


 n  n+1
1 1 2pn Pn
pn = 1 + , Pn = 1 + , hn = .
n n pn + Pn
Prove that h1 < h2 < h3 < . . .

(B5) Show that ⌈( 3 + 1)2n )⌉ is divisible by 2n+1 .

(B6) A particle moves on a circle with center O, starting from rest at a point P
and coming to rest again at a point Q, without coming to rest at any interme-
diate point. Prove that the acceleration vector of the particle does not vanish
at any point between P and Q and that, at some point R between P and Q,
the acceleration vector points in along the radius RO.

1942
(A1) A square of side 2a, lying always in the first quadrant of the xy-plane,
moves so that two consecutive vertices are always on the x- and y-axes respec-
tively. Find the locus of the midpoint of the square.

(A3) Is the series


∞  n
X n! 19
·
n=0
(n + 1)n 7
convergent or divergent?

(A4) Find the orthogonal trajectories of the family of conics (x+2y)2 = a(x+y).
At what angle do the curves of one family cut the curves of the other family at
the origin?

Page 98
Putnam In Olympiad Mahdiyar Amid Sardari

(A5) A circle of radius a is revolved through 180◦ about a line in its plane,
distant b from the center of the circle, where b > a. For what value of the ratio
b
a does the center of gravity of the solid thus generated lie on the surface of the
solid?

(A6) Any circle in the xy-plane is ”represented” by a point on the vertical


line through the center of the circle and at a distance ”above” the plane of the
circle equal to the radius of the circle. Show that the locus of the representations
of all the circles which cut a fixed circle at a constant angle is a portion of a
one-sheeted hyperboloid. By consideration of a suitable family of circles in the
plane, demonstrate the existence of two families of rulings on the hyperboloid.

(B1) A square of side 2a, lying always in the first quadrant of the xy-plane,
moves so that two consecutive vertices are always on the x- and y-axes respec-
tively. Prove that a point within or on the boundary of the square will in general
describe a portion of a conic. For what points of the square does this locus de-
generate?

(B2) For the family of parabolas

a3 x2 a2 x
y= + − 2a
3 2

(1) find the locus of vertices,


(2) find the envelope,
(3) sketch the envelope and two typical curves of the family.

(B3) Given x = ϕ(u, v) and y = ψ(u, v), where ϕ and ψ are solutions of the
partial differential equation
∂ϕ ∂ψ ∂ϕ ∂ψ
(1) − = 1.
∂u ∂v ∂v ∂u
By assuming that x and y are the independent variables, show that (1) may be
transformed to
∂y ∂u
(2) = .
∂v ∂x
Integrate (2) and show how this effects in general the solution of (1). What
other solutions does (1) possess?

(B4) A particle moves under a central force inversely proportional to the k-


th power of the distance. If the particle describes a circle ( the central force
proceeding from a point on the circumference of the circle ), find k.

Page 99
Putnam In Olympiad Mahdiyar Amid Sardari

1941
(A1) Prove that the polynomial:
5 1
(a − x)6 − 3a(a − x)5 + a2 (a − x)4 − a4 (a − x)2
2 2
takes only negative values for 0 < x < a.

(A3) A circle of radius a rolls in the plane along the x-axis. Show that the
envelope of a diameter is a cycloid, coinciding with the cycloid traced out by a
point on the circumference of a circle of diameter a, likewise rolling in the plane
along the x-axis.

(A5) The line L is parallel to the plane y = z and meets the parabola 2x =
y 2 , z = 0 and the parabola 3x = z 2 , y = 0. Prove that if L moves freely
 subject
to these constraints then it generates the surface x = (y − z) y2 − z3 .

(B4) Given two perpendicular diameters AB and CD of an ellipse, we say


that the diameter A′ B ′ is conjugate to AB if A′ B ′ is parallel to the tangent to
the ellipse at A. Let A′ B ′ be conjugate to AB and C ′ D′ be conjugate to CD.
Prove that the rectangular hyperbola through A′ , B ′ , C ′ and D′ passes through
the foci of the ellipse.

1940
(A1) Prove that if f (x) is a polynomial with integer coefficients and there exists
an integer k such that none of f (1), . . . , f (k) is divisible by k, then f (x) has no
integral root.

(A2) Let A, B be two fixed points on the curve y = f (x), f is continuous


with continuous derivative and the arc AB
d is concave to the chord AB. If P is
a point on the arc AB for which AP + P B is maximal, prove that P A and P B
d
are equally inclined to the tangent to the curve y = f (x) at P .

(A4) Let p be a real constant. The parabola y 2 = −4px rolls without slip-
ping around the parabola y 2 = 4px. Find the equation of the locus of the
vertex of the rolling parabola.

(A5) Prove that the simultaneous equations

x4 − x2 = y 4 − y 2 = z 4 − z 2

are satisfied by the points of 4 straight lines and 6 ellipses, and by no other
points.

Page 100
Putnam In Olympiad Mahdiyar Amid Sardari

P∞ P∞
(A7) If i=1 u2i and i=1 vi2 are convergent series of real numbers, prove that

X
(ui − vi )p
i=1

is convergent, where p ≥ 2 is an integer.

(B3) Let p > 0 be a real constant. From any point (a, b) in the Cartesian
plane, show that
i1) Three normals, real or imaginary, can be drawn to the parabola y 2 = 4px.
2) These are real and distinct if 4(2 − p)3 + 27pb2 < 0.
3) Two of them coincide if (a, b) lies on the curve 27py 2 = 4(x − 2p)3 .
4) All three coincide only if a = 2p and b = 0.

(B4) Prove that the locus of the point of intersection of three mutually per-
pendicular planes tangent to the surface

ax2 + by 2 + cz 2 = 1 (where abc ̸= 0)

is the sphere
1 1 1
x2 + y 2 + z 2 = + + .
a b c

(B5) Suppose that the rational numbers a, b and c are the roots of the equation
x3 + ax2 + bx + c = 0. Find all such rational numbers a, b and c. Justify your
answer.

1939
(A2) Let C be the curve y = x3 (where x takes all real values). The tangent
at A meets the curve again at B. Prove that the gradient at B is 4 times the
gradient at A.

(A3) The roots of x3 + ax2 + bx + c = 0 are α, β and γ. Find the cubic whose
roots are α3 , β 3 , γ 3 .

(A4) Given 4 lines in Euclidean 3−space:


L1 : x = 1, y = 0;
L2 : y = 1, z = 0;
L3 : x = 0, z = 1;
L4 : x = y, y = −6z.
Find the equations of the two lines which both meet all of the Li .

Page 101
Putnam In Olympiad Mahdiyar Amid Sardari

(A6) Do either (1) or (2):


(1) A circle radius r rolls around the inside of a circle radius 3r, so that a point
on its circumference traces out a curvilinear triangle. Find the area inside this
figure.
(2) A frictionless shell is fired from the ground with speed v at an unknown
angle to the vertical. It hits a plane at a height h. Show that the gun must be
1
sited within a radius vg (v 2 −2gh) 2 of the point directly below the point of impact.

(B3) Given an = (n2 + 1)3P


n
, find a recurrence relation an + pan+1 + qan+2 +
ran+3 = 0. Hence evaluate n≥0 an xn .

1938
(A1) A solid in Euclidean 3-space extends from z = −h +h
2 to z = 2 and the area
of the section z = k is a polynomial in k of degree at most 3. Show that the
volume of the solid is h(B+4M
6
+T )
, where B is the area of the bottom (z = −h2 ),
M is the area of the middle section (z = 0), and T is the area of the top (z = h2 ).
Derive the formulae for the volumes of a cone and a sphere.

(A2) A solid has a cylindrical middle with a conical cap at each end. The
height of each cap equals the length of the middle. For a given surface area,
what shape maximizes the volume?

(A4) A notch is cut in a cylindrical vertical tree trunk. The notch penetrates
to the axis of the cylinder and is bounded by two half-planes. Each half-plane
is bounded by a horizontal line passing through the axis of the cylinder. The
angle between the two half-planes is θ. Prove that the volume of the notch is
minimized (for given tree and θ) by taking the bounding planes at equal angles
to the horizontal plane.

(A6) A swimmer is standing at a corner of a square swimming pool. She swims


at a fixed speed and runs at a fixed speed (possibly different). No time is taken
entering or leaving the pool. What path should she follow to reach the opposite
corner of the pool in the shortest possible time?

(A7) Do either (1) or (2)


(1) S is a thin spherical shell of constant thickness and density with total mass
M and center O. P is a point outside S. Prove that the gravitational attraction
of S at P is the same as the gravitational attraction of a point mass M at O.
(2) K is the surface z = xy in Euclidean 3−space. Find all straight lines lying
in S. Draw a diagram to illustrate them.

Page 102
Putnam In Olympiad Mahdiyar Amid Sardari

(B4) The parabola P has focus a distance m from the directrix. The chord
AB is normal to P at A. What is the minimum length for AB?

(B5) Find the locus of the foot of the perpendicular from the center of a rect-
angular hyperbola to a tangent. Obtain its equation in polar coordinates and
sketch it.

(B6) What is the shortest distance between the plane Ax + By + Cz + 1 = 0 and


2 2 2
the ellipsoid xa2 + yb2 + zc2 = 1. You may find it convenient to use the notation
−1 1
h = (A2 + B 2 + C 2 ) 2 , m = (a2 A2 + b2 B 2 + c2 C 2 ) 2 . What is the algebraic
condition for the plane not to intersect the ellipsoid?

Page 103

You might also like